nurse

Published on March 2017 | Categories: Documents | Downloads: 68 | Comments: 0 | Views: 1054
of 89
Download PDF   Embed   Report

Comments

Content

1. Mode of transmission of Pertussis: (a) Vector transmission (b) Through contaminated water (c) Droplet infection (d) Faecoaral 2. Time required to measure the Mantaux test: (a) 24—48 hrs (b) 48—72 hrs (c) 10—12 hrs (d) 2—4 hrs 3. Name of the oral Polio Vaccine: (a) BCG (b) TAB (c) Salk (d) Sabin 4. The following are the standard tuberculosis drug except: (a) Chloromycin (b) Streptomycin (c) INH (d) PAS 5. Combination of Tripple Antigen: (a) DPT (b) BCG (c) TAB (d) ECG 6. The time interval between infection of suspected person and the appearance of signs and symptom of the disease is called (a) Quarrentine (b) Recovery period (c) Incubation (d) Isolation period 7. Route of transmission of Hepatitis B: (a) Oral (b) Potential (c) Faeco oral (d) Contaminated water 8. Mode of transmission of typhoid fever: (a) Intravenous (b) Oro faceal (c) Respiratory (d) Droplet 9. Causative organism of diphtheria: (a) Mycobacteria (b) Streptococcus (c) Coryne Bacterium diphtheria (d) Staphylococcus 10. Incubation period of measles: (a) 1—7 days (b) 7—14 days (c) 14—21 days (d) 21—28 days 11. Site for mantaux test: (a) Outer and upper quadrant of gluteus maximus (b) Deltoid region (c) Posterior left forearm (d) Anterior left forearm 12. Media of transmission of bovine type of tubercle bacilli: (a) Milk (b) Water (c) Air (d) Food 13. Painful Micturition: (a) Eneuresis (b) Dysuria (c) Rentention of urine (d) Diarrhoea 14. Amount of solution used for evacuant enema in adults: (a) 100—250 ml (b) 250—350ml (c) 500—1000ml (d) 1000—1500 ml 15. Drugs used to relieve pain: (a) Anti inflammatory (b) Antipyretics (c) Antibiotics (d) Analgesics 16. Hormone secreted by the testes: (a) Oestrogen (b) Pitocin (c) Androgens (d) Progesterone 17. One teaspoon contains ............. ml (a) 2ml (b) 5ml (c) 10ml (d) 15ml 18. One ml contains ........... drops (a) 5 drops (b) 10 drops (c) 15 drops (d) 20 drops

19. Average weight of a new born baby: (a) 2 kg (b) 4 Kg (c) 3kg (d) 5kg 20. How much salt must be added to one litre of water to make a solution of normal saline? (a) 2 grams (b) 5 grams (c) 6 grams (d) 9 grams 21. Number of vertebra in the sacrum: (a) 5 (b) 3 (c) 4 (d) 1 22. Number of chromosome in a cell: (a) 23 (b) 44 (c) 46 (d) 40 23. A trial septal defect occurs due to the persistence of: (a) Formen ovale (b) Ductus arteriosis (c) Ductus venosus (d) Mitral valve 24. .......... is a water soluble vitamin: (a) Vitamin A (b) Vitamin K (c) Vitamin B (d) Vitamin D 25. Quickening means: (a) Hearing of the fetal heart sound (b) Sudden cessation of Menstruation (c) The movement of the feuts in the uterus felt by the mother (d) Nausa and vomiting in the morning 26. Cholera is caused by a kind of bacterium called: (a) Tubercle bacillus (b) Vibrio cholerae (c) Protozoa (d) Entamoeba histolytica 27. Drinking of alcohol will severely effects the (a) Kidney (b) Brain (c) Stomach (d) Liver 28. The disease in which the patients blood does not clot easily is: (a) Sickle cell anaemia (b) Haemophilia (c) Rabies (d) Diabetes 29. Discolouration of teeth is caused by a kind of antibiotics called (a) Streptomycin (b) Neomycin (c) Tetracycline (d) Penicillin 30. ........... is caused by the fungus (a) Malaria (b) Athletes foot (c) Filariasis (d) Typhus fever 31. The Filariasis is transmitted by ........ (a) Anopheles female mosquito (b) Anopheles male mosquito (c) Culex female mosquito (d) Culex male mosquito 32. The blood group contain no antibodies: (a) A (b) B (c) O (d) AB 33. The disease that spread through sexual contact is .......... (a) Leprosy (b) AIDS (c) Itches (d) Scabies 34. Typhoid is transmitted through -(a) Water and food (b) Air (c) Touch (d) Personal contact 35. ............ is example for the disease caused by bacteria: (a) Cold (b) Typhoid (c) Typhus fever (d) Chicken pox 36. ......... is the immunity get by the body after the first attack of disease. (a) Natural immunity (b) Passive immunity (c) Artificial immunity (d) Acquired immunity 37. The cells that are responsible for the production of antibodies are .......

(a) Red blood cells (b) Platelets (c) Plasma (d) White blood cells 38. A healthy person can donate blood ...... or ..... times a year. (a) Six or seven (b) Five or six (c) Three or four (d) Two or three 39. Radio active cobalt and radium are used for the treatment of --diseases. (a) Cholera (b) Haemophilia (c) Cancer (d) Sickle cell anaemia 40. World health day is on ....... (a) May 12th (b) April 7th (c) April 30th (d) December 18th 41. To relieve from dyspnoea the appropriate position to be used is: (a) Prone position (b) Supine position (c) Side lying position (d) Fowler's position 42. Protein requirement for a pregnant woman per day: (a) 50grams (b) 55 grams (c) 60 grams (d) 65 grams 43. The findus is at the level of umbilicus during .......... week pregnancy (a) 16 (b) 20 (c) 24 (d) 28 44. Total weight gain during pregnancy is (a) 7—8 kg (b) 10—11 kg (c) 13—14 kg (d) 15—16 kg 45. The birth weight of the baby doubles at the age of (a) 3 months (b) 6months (c) 9months (d) 12months 46. Infancy period is from (a) Birth to one month (b) One month to 1 year (c) 1 years to 3 years (d) 3 years to 5 years 47. The following are the signs of dehydration except: (a) Sunken tearless eyes (b) Loss of skin elasticity (c) Diuresis (d) depressed fintenelle 48. The following are the contests of ORS except: (a) Sodium chloride (b) Sugar (c) Potassium Chloride (d) Calcium Carbonate 49. The most common from of malnutrition are the following except (a) Beri-beri (b) Protein energy malnutrition (c) Marasmus (d) Kwashiorkor 50. The first course of oral pills should be stared on the ..... day of menstrual cycle. (a) 2nd (b) 7th (c) 5th (d) 10th 51. The following are the natural method of contraception except: (a) Foam tablet (b) Basal metabolic temperature (c) Rhythm method (d) Safe period 52. ......... is the permanent method of sterilization. (a) MTP (b) Diaphragm (c) Tubectomy (d) D & C 53. The following organs are situated in the abdominal cavity except: (a) Stomach (b) Oesophagus (c) liver (d) Spleen 54. The system of body which helps for the removal of waste matter from body: (a) Digestive system (b) Endocrine system (c) Respiratory system (d) Excretory system 55. Total number of bones in human body: (a) 306 (b) 206 (c) 106 (d) 406 56. Eruption of teeth starts at the age of ....... month (a) 4 months (b) 6 months (c) 8 months (d) 10 months

57. The never, which carries messages from the brain and spinal cord muscles and glands: (a) Motor nerve (b) Sensory nerve (c) Mixed nerve (d) Autonomic nerve 58. Speech centre is situated in: (a) Perietal lobe (b) Frontal lobe (c) Occipital lobe (d) Temporal lobe 59. Largest gland in the body: (a) Pituitary gland (b) Adrenal gland (c) Liver (d) Gall bladder 60. Head control of the infant occurs at the age of ...... month (a) 2 (b) 3 (c) 5 (d) 6 61. Peurparium is the period begins: (a) As soon as the placenta is expelled and last for 6 to 8 weeks (b) As soon as the baby is expelled and last for 6 to 8 weeks (c) As soon as the membrane ruptures and last for 6 to 8 weeks (d) As soon as the placenta is expelled and last for 10 to 12 weeks 62. When the umbilical cord lies in front of the presenting part and the membranes are intact it is known as: (a) Cord prolapse (b) Cord presentation (c) Cord pulsation (d) Cord delivery 63. Starting of menstrual cycle is called: (a) Ovulation (b) Menstruation (c) Menarche (d) Menopause 64. Normal blood urea level is: (a) 40—60 mg% (b) 20—40 mg% (c) 10—20 mg% (d) 80—120 mg% 65. One gram of carbohydrate yields: (a) 4 calories (b) 8 calories (c) 9 calories (d) 1 calorie 66. A method of making the victim to breath passively (a) Artificial respiration (b) Spontaneous respiration (c) Deep breathing (d) Kusmal breathing 67. Minute, this walled blood vessels between the ends of the arteries and beginning of veins is called: (a) Arteries (b) Veins (c) Capillaries (d) Alveoli 68. A waste gas produced by the body and exhaled through the lungs is: (a) Carbon dioxide (b) Carbon monoxide (c) Nitrogen peroxide (d) Oxygen 69. A condition characterized by moving of bones out joint: (a) Fracture (b) Sprain (c) Strain (d) Dislocation 70. A tube of muscular tissue carrying ingested food from the mouth to stomach: (a) Oesophagus (b) Duodenum (c) Trachea (d) Pharynx 71. A red pigment in the blood cells which combines with oxygen and carbon dioxide for carrying them: (a) Red blood cells (b) Haemoglobin (c) Platelet (d) Plasma 72. Information given by the sufferer about his illness: (a) Sign (b) Symptom (c) Observation (d) Diagnosis 73. Normal respiratory rate in adults is: (a) 20—40/min (b) 40—60/min (c) 16—20/min (d) 10—15/min 74. The normal body temperature is ........ 0C. (a) 350C (b) 370C (c) 400C (d) 420C

75. Characteristic of the blood flow due to arterial bleeding is the following except: (a) Bright red in colour (b) Spunts at each cartrachian of the heart (c) Flow is pulsatile (d) Dark red in colour 76. An agent that has power to kill Micro-organism: (a) Bacteriostat (b) Bacteriocide (c) Antiseptic (d) Bacteriostasis 77. Strength of savlon forcleaning of wounds: (a) 1:10 (b) 1:100 (c) 1:1000 (d) 1:10000 78. Inflammation of the tongue: (a) Glossitis (b) Gingivitis (c) Stomatitis (d) Parotitis 79. Following are predisposing cause for bedsore except: (a) Impaired circulation (b) Lowered vitality (c) Emaciation (d) Anorexia 80. Inability to sleep is termed as: (a) Dyspepsia (b) Dyspnaea (c) Anorexia (d) Insumania 81. Tobacco contains a poisonous substance called (a) Nicotine (b) Narcotics (c) Alcohol (d) Caffeine 82. A disease caused by allergic disease is called (a) Asthma (b) Tuberculosis (c) Cancer (d) Gastroenteritis 83. An adult has ...... litres of blood in his body. (a) 7-8 (b) 5-6 (c) 9-10 (d) 2-3 84. Several persons in a town got the attack of leukemia. Which of the following can be possible reason for that? (a) Exposed to radiation (b) Drinking polluted water (c) Smoking (d) Breathing in impure air 85. The energy value of ......... is more than that of others (a) Fats (b) Starch (c) Protein (d) Sugar 86. The product formed when amino acid molecules combine together is called (a) Nucleic acid (b) Starch (c) Carbohydrate (d) Proteins 87. Women having normal limbs may sometimes give birth to babies with deformed limbs. This must be due to: (a) Spontaneous generation (b) Mutation (c) Inheritance of acquired character (d) Natural selection 88. Hormones are transported to all parts of the body through the (a) Nerves (b) Blood (c) Lymph (d) Muscles 89. Deficiency of ....... in food causes simple goiter. (a) Sodium (b) Iron (c) Iodine (d) Calcium 90. Cortisone is used as an effective anti-inflamatory drug in the treatment of .......... (a) High blood pressure (b) Arteroisclerosis (c) Arthritis (d) Diabetes 91. Deficiency of thyroxin in adults leads to a condition called: (a) Tetani (b) Cretinism (c) Myxoedema (d) Graves diseases 92. The hormone that is injected to pregnant women at the time of delivery is (a) Vasopressin (b) Oxytocin (c) Androgen (d) Oestrogen

93. The outer layer of the eye ball is (a) Sclera (b) Choroids (c) Retina (d) Conjunctiva 94. The smallest bone in the human body is (a) Malleus (b) Incus (c) Stapes (d) Sternum 95. Central nervous system includes: (a) The brain and cranial nerves (b) The cranial nerves and spinal cord (c) The spinal nerves and brain (d) The spinal nerves and brain 96. Short sightedness can be corrected by using spectacles with ..... lens. (a) Concave (b) Convex (c) Cylindrical (d) Opaque 97. Normal sugar level in our blood is: (a) 95—125 mg/100ml (b) 65—130 mg/100ml (c) 80—120 mg/100ml (d) 90—140 mg/100 ml 98. Labour takes place after ...... days of last menstrual period (a) 300 (b) 280 (c) 365 (d) 240 99. Signs of true labour are the following except: (a) Painful rhythmic uterine contraction (b) Dilatation of the OS (c) Fetal movement (d) Show 100. Signs of separation of placenta are the following except: (a) Lengthening of the cord at vulva (b) Gush of blood is seen (c) Fundus risas upto umbilicus (d) Temperature rises Answers:

1 C 11 D 21 A 31 C 41 D 51 A 61 A 71 B 81 A 91 C 2 B 12 A 22 C 32 D 42 B 52 C 62 B 72 B 82 A 92 B 3 D 13 B 23 A 33 B 43 C 53 B 63 C 73 C 83 B 93 D 4 A 14 C 24 C 34 A 44 C 54 D 64 B 74 B 84 A 94 C 5 A 15 D 25 C 35 B 45 B 55 B 65 A 75 D 85 A 95 C 6 C 16 C 26 B 36 D 46 B 56 B 66 A 76 B 86 D 96 A 7 B 17 B 27 D 37 D 47 C 57 A 67 C 77 A 87 B 97 C 8 B 18 C 28 B 38 C 48 D 58 A 68 A 78 A 88 B 98 B 9 C 19 C 29 C 39 C 49 A 59 C 69 D 79 D 89 C 99 C 10 B 20 D 30 B 40 B 50 C 60 C 70 A 80 D 90 C 100 D

NURSING - OPTIONAL MODEL QUESTIONS : CLASS – XII Time – 3 hrs Marks - 150 Section A – Answer all questions Section B – Answer any 15 questions Section C – Questions 71 is compulsory and answer any five from the remaining questions Section D – Answer any four questions in 200 words Section - A Answer all questions I. Choose the Following 10 x 1 = 10 1. Disease caused by viruses are a) Diphtheria b) Tetanus c) Pertussis d) Measles 2. Mumps is a virus infection. It affects the a) Parotid gland b) Pituitary gland c) Thyroid gland d) Adrenal gland 3. Cholera is an acute communicable disease caused by a) Bacilli b) Virus c) Mycobacterium d) Vibriyo 4. Infection of hepatitis affects the a) Spleen b) Stomach c) Liver d) Intestine 5. Calories are the unit to measure a) Growth b) Heat c) Energy d) Development 6. Vitamin B12 is also needed for producing a) White blood cells b) Red blood cells c) Plasma d) None of these 7. Capsules are made up of a) Tablets b) Gelatin c) Liquid d) Powder 8. Fracture in which one side of a bone is broken, and the other side is bent is called a) Compound fracture b) Simple fracture c) Greenstick fracture d) Depressed fracture 9. Unconscious is a condition in which there is depression of cerebral function called a) stupor of coma b) convulsions

c) concussion d) intracranial hemorrhage 10. This method is suitable for enamel, metal glass a) boiling b) filtering c) chemicals d) dry heat II. State whether the following statements are True or False 10 x 1 = 10 11. Water borne, food borne diseases are measles and common cold. 12. Sign and symptom of typhoid fever is high fever more than a weak. 13. Protein deficiency is called marasmus. 14. Sources of carbohydrates are Rice, wheat, cereals and root vegetables. 15. Bacteria causing diseases are cholera, Typhoid, Dysentery. 16. ‘Vibrio’ BACILLI cause Typhoid fever. 17. INFLAMMATION of the tongue is called stomatitis. 18. Glossitis means inflammation of the mouth. 19. Kwashiorker is due to deficiency of carbohydrates. 20. Disinfection is the process of killing the pathogenic micro-organisms. III. Fill in the blanks 20 x 1 = 20 21. Till 1950 _____________ was considered to be a major health problem in India. 22. _____________ by 1990 was part of a global effort, coordinated by the world health organization. 23. In every school a fully equipped _____________ box should be available at hand. 24. _____________ is the back bone of public health Nursing. 25. National AIDS control programme was launched is INDIA in theyear _____________

26. In English “The Word” midwife means _____________ (the person with the women who is in labour)

27. _____________ is a passage of menstrual flow and is the exit of the foetus during delivery. 28. The ovaries produce ovum and the female hormones _____________ 29. _____________ produces a thin lubricating fluid which enters the urethra through ducts. 30. _____________ is primarily responsible for stimulating the ovaries to secrete corpusleuteum. 31. _____________ is primarily responsible for stimulating the corpusleuteum for production of progesterone. 32. _____________ is the end of a woman reproductive. 33. In _____________ the foetus floats. 34. _____________ is important for the promotion of adequate foetal growth. 35. _____________ is the removal of urine by means of catheter. 36. _____________ Operation involves a large surface area of the body. 37. _____________ Organisms are those which produce disease. 38. _____________ are plant like organisms. 39. _____________ are animal type of organisms. 40. _____________ are smaller than bacteria. IV. Match the following 10 x 1 = 10 41. Protozoa - Plant type of organisms

42. Fungi - Rod shaped organisms 43. Bacilli - Triple antigen 44. B.C.G. - Cholera 45. D.P.T. - Animal type of organisms 46. Epidemic - Vaccine against tuberculosis 47. Mosquito - The germs live in a soil and cowdung 48. Air borne disease - Malaria 49. Tetanus - Water borne disease 50. Typhoid - Common cold, measles PART - B V. Answer the fifteen questions in two lines each 15 x 2 = 30 51. What are the reasons for majority of death and illness? 52. What is infection? 53. What are all the diseases spread by droplet infection? 54. Write two signs and symptoms of upper respiratory infections? 55. What is the complication of mumps? 56. What is poliyomylitis? 57. What is Asepsis? 58. What are the nutrients materials present in food? 59. What are the staple food? 60. What are the sources of proteins?

61. What is the benefits of vitamin A? 62. What are the sources and deficiency of Iron? 63. What do you mean by dietary food? 64. Define medication. 65. What is Analgesic? 66. What is Antidotes? 67. What are the reasons for Paratiditis? 68. What is gastritis? 69. What is appendicitis? 70. What is constipation? PART - C VI. Answer any five questions in 100 words each from 72 - 77 (Question No. 71 is compulsory) 6 x 5 = 30 71. What are the principles of administering medicines? ( or ) Write the wars of administration of the drugs? 72. What are the types of hepatitis? 73. Write the signs and symptoms of tuberculosis? 74. What are the types of fracture? 75. Write about the immunization schedule? 76. What are the common accidents in children?

77. Write the deficiency of proteins? PART - D VII. Answer any four questions in 200 words each 4 x 10 = 40 78. Write the principles of isolation technique? 79. How will you handle the sterile articles? 80. Describe about the sterilizing methods? 81. Write the differences between good nutrition and malnutrition? 82. Write the procedure of Ryles tube feeding? 83. What are the side effects of drugs?

Nursing Aptitude Test Question No.1 If acceleration, due to gravity at earth, is ‘g’ and mass of earth is 80 times that of moon and radius of earth is 4 times that of moon, the value of ‘g’ at surface of moon will be: a) g b) g/20 c) g/5 d) 320g Answer Let M and R be the mass and radius of earth, M’ and R’ be the mass and radius of moon. Then R’ = R/4 and M’=M/80. Let g and g’ be the acceleration of gravity on the surface of earth and moon respectively. Then g=GM / R*R g’=GM’ / R’*R’ = G * ( M/80) / ((R/4)* (R/4)) = g/5 The correct answer is g/5.

Nursing Aptitude Test Question No.2 What are iatrogenic infections? Answer These are procedural infections i.e. infections that are caused by procedures. The main example is bacterial infection after IV insertion. Nursing Aptitude Test Question No.3 What are meanings of the following words? a) Chary b) Insipid c) Stanch d) Fallacious e) Inimical f) Ebullient Answer a) Cautious b) Tasteless food c) To stop the flow of blood from a wound d) Based on error or misleading e) Harmful f) Enthusiasm Nursing Aptitude Test Question No.4 The smallest number which when diminished by 3 is divisible by 21, 28,36 and 45 is: a) 423 b) 1257 c) 1263

d) 1260 Answer 1263 Nursing Aptitude Test Question No.5 A cone, hemisphere and a cylinder stand on equal base and have same height. The ratio of their volumes is a) 1:2:3 b) 2:1:3 c) 2:3:1 d) 3:2:1 Answer Try to find out your answer. Note: These practice questions are designed just to help you understanding the general style of the nursing aptitude tests. However, to minimize your chance of rejection for your dreamed nursing career, you need to practice a couple of such tests online or offline. This practice shall give you familiarity with general structure of the tests and help you to qualify with ease.

NEW Free NCLEX-RN Sample Test Questions For Nursing Review (Part 16)
Question 1 The nurse prepares for a Denver Screening of a 3 year-old child in the clinic. The mother asks the nurse to explain the purpose of the test. What is the nurse’s best response about the purpose of the Denver? A) "It measures a child’s intelligence." B) "It assesses a child's development." C) "It evaluates psychological responses." D) " It helps to determine problems."

Review Information: The correct answer is B: "It assesses a child''s development."

The Denver Developmental Test II is a screening test to assess children from birth through 6 years in personal/social, fine motor adaptive, language and gross motor development. A child experiences the fun of play during the test. Question 2 In planning care for a child diagnosed with minimal change nephrotic syndrome, the nurse should understand the relationship between edema formation and A) increased retention of albumin in the vascular system B) decreased colloidal osmotic pressure in the capillaries C) fluid shift from interstitial spaces into the vascular space D) reduced tubular reabsorption of sodium and water Review Information: The correct answer is B: decreased colloidal osmotic pressure in the capillaries The increased glomerular permeability to protein causes a decrease in serum albumin, which results in decreased colloidal osmotic pressure.

Question 3 Based on principles of teaching and learning, what is the best initial approach to pre-op teaching for a client scheduled for coronary artery bypass? A) Touring the coronary intensive unit B) Mailing a video tape to the home C) Assessing the client's learning style D) Administering a written pre-test Review Information: The correct answer is C: Assessing the client''s learning style As with any anticipatory teaching, assess the client''s level of knowledge and learning style first. Question 4 A client is admitted with a diagnosis of hepatitis B. In reviewing the initial laboratory results, the nurse would expect to find elevation in which of the following values? A) Blood urea nitrogen B) Acid phosphatase C) Bilirubin D) Sedimentation rate

Review Information: The correct answer is C: Bilirubin In the laboratory data provided, the only elevated level expected is bilirubin. Additional liver function tests will confirm the diagnosis.

Question 5 The nurse is caring for a toddler with atopic dermatitis. The nurse should instruct the parents to A) Dress the child warmly to avoid chilling B) Keep the child away from other children for the duration of the rash C) Clean the affected areas with tepid water and detergent D) Wrap the child's hand in mittens or socks to prevent scratching

Review Information: The correct answer is D: Wrap the child''s hand in mittens or socks to prevent scratching A toddler with atopic dermatitis needs to have fingernails cut short and covered so the child will not be able to scratch the skin lesions, thereby causing new lesions and possibly a secondary infection. Question 6 The nurse is planning to give a 3 year-old child oral digoxin. Which of the following is the best approach by the nurse? A) "Do you want to take this pretty red medicine?" B) "You will feel better if you take your medicine." C) "This is your medicine, and you must take it all right now." D) "Would you like to take your medicine from a spoon or a cup?" Review Information: The correct answer is D: "Would you like to take your medicine from a spoon or a cup?" At 3 years of age, a child often feels a loss of control when hospitalized. Giving a choice about how to take the medicine will allow the child to express an opinion and have some control.

Question 7 Which of the actions suggested to the registered nurse (RN) by the practical nurse (PN) during a planning conference for a 10 month-old infant admitted 2 hours ago with bacterial meningitis would be acceptable to add to the plan of care? A) measure head circumference B) place in airborne isolation C) provide passive range of motion D) provide an over-the-crib protective top

Review Information: The correct answer is A: measure head circumference In meningitis, assessment of neurological signs should be done frequently. Head circumference is measured because subdural effusions and obstructive hydrocephalus can develop as a complication of meningitis. The client will have already been on airborne precautions and crib top applied to the bed on admission to the unit. Question 8 During the evaluation phase for a client, the nurse should focus on A) All finding of physical and psychosocial stressors of the client and in the family B) The client's status, progress toward goal achievement, and ongoing reevaluation C) Setting short and long-term goals to insure continuity of care from hospital to home D) Select interventions that are measurable and achievable within selected timeframes Review Information: The correct answer is B: The client''s status, progress toward goal achievement, and ongoing re-evaluation The evaluation step of the nursing process focuses on the client''s status, progress toward goal achievement and ongoing re-evaluation of the plan of care. The other possible answers focus on other steps of the nursing process.

Question 9

The nurse would expect the cystic fibrosis client to receive supplemental pancreatic enzymes along with a diet A) high in carbohydrates and proteins B) low in carbohydrates and proteins C) high in carbohydrates, low in proteins D) low in carbohydrates, high in proteins

Review Information: The correct answer is A: high in carbohydrates and proteins Provide a high-energy diet by increasing carbohydrates, protein and fat (possibly as high as 40%). A favorable response to the supplemental pancreatic enzymes is based on tolerance of fatty foods, decreased stool frequency, absence of steatorrhea, improved appetite and lack of abdominal pain. Question 10 The nurse enters a 2 year-old child's hospital room in order to administer an oral medication. When the child is asked if he is ready to take his medicine, he immediately says, "No!". What would be the most appropriate next action? A) Leave the room and return five minutes later and give the medicine B) Explain to the child that the medicine must be taken now C) Give the medication to the father and ask him to give it D) Mix the medication with ice cream or applesauce Review Information: The correct answer is A: Leave the room and return five minutes later and give the medicine Since the nurse gave the child a choice about taking the medication, the nurse must comply with the child''s response in order to build or maintain trust. Since toddlers do not have an accurate sense of time, leaving the room and coming back later is another episode to the toddler.

Question 11 A 4 year-old child is recovering from chicken pox (varicella). The parents would like to have the child return to day care as soon as possible. In order to ensure that the illness is no longer communicable, what should the nurse assess for in this child? A) All lesions crusted B) Elevated temperature C) Rhinorrhea and coryza D) Presence of vesicles

Review Information: The correct answer is A: All lesions crusted The rash begins as a macule, with fever, and progresses to a vesicle that breaks open and then crusts over. When all lesions are crusted, the child is no longer in a communicable stage. Question 12 The nurse is providing instructions to a new mother on the proper techniques for breast feeding her infant. Which statement by the mother indicates the need for additional instruction? A) "I should position my baby completely facing me with my baby's mouth in front of my nipple." B) "The baby should latch onto the nipple and areola areas." C) "There may be times that I will need to manually express milk." D) " I can switch to a bottle if I need to take a break from breast feeding." Review Information: The correct answer is D: " I can switch to a bottle if I need to take a break from breast feeding." Babies adapt more quickly to the breast when they are not confused about what is put into their mouths and its purpose. Artificial nipples do not lengthen and compress the way the human nipples (areola) do. The use of an artificial nipple weakens the baby''s suck as the baby decreases the sucking pressure to slow fluid flow. Babies should not be given a bottle during the learning stage of breast feeding.

Question 13 A victim of domestic violence tells the batterer she needs a little time away. How would the nurse expect that the batterer might respond? A) With acceptance and views the victim’s comment as an indication that their marriage is in trouble B) With fear of rejection causing increased rage toward the victim C) With a new commitment to seek counseling to assist with their marital problems D) With relief, and welcomes the separation as a means to have some personal time

Review Information: The correct answer is B: With fear of rejection causing increased rage toward the victim

The fear of rejection, abandonment, and loss only serve to increase the batterer’s rage at the partner. Question 14 The nurse, assisting in applying a cast to a client with a broken arm, knows that the A) cast material should be dipped several times into the warm water B) cast should be covered until it dries C) wet cast should be handled with the palms of hands D) casted extremity should be placed on a cloth-covered surface Review Information: The correct answer is C: wet cast should be handled with the palms of hands Handle cast with palms of the hands and lift at 2 points of the extremity. This will prevent stress at the injury site and pressure areas on the cast.

Question 15 A recovering alcoholic asked the nurse, "Will it be ok for me to just drink at special family gatherings?" Which initial response by the nurse would be best? A) "A recovering person has to be very careful not to lose control, therefore, confine your drinking only to family gatherings." B) "At your next AA meeting discuss the possibility of limited drinking with your sponsor." C) "A recovering person needs to get in touch with their feelings. Do you want a drink?" D) "A recovering person cannot return to drinking without starting the addiction process over." Review Information: The correct answer is D: "A recovering person cannot return to drinking without starting the addiction process over." Recovery requires total abstinence from all drugs. Question 16 The nurse is assessing a child for clinical manifestations of iron deficiency anemia. Which factor would the nurse recognize as the cause of the findings? A) Decreased cardiac output B) Tissue hypoxia C) Cerebral edema

D) Reduced oxygen saturation

Review Information: The correct answer is B: Tissue hypoxia When the hemoglobin falls sufficiently to produce clinical manifestations, the findings are directly attributable to tissue hypoxia, resulting from a decrease in the oxygen carrying capacity of the blood.

Question 17 A nurse is assigned to a client who is newly admitted for treatment of a frontal lobe brain tumor. Which history offered by the family members would be recognized by the nurse as associated with the diagnosis, and communicated to the provider? A) "My partner's breathing rate is usually below 12." B) "I find the mood swings and the change from a calm person to being angry all the time hard to deal with." C) "It seems our sex life is nonexistent over the past 6 months." D) "In the morning and evening I hear complaints that reading is next to impossible from blurred print."

Review Information: The correct answer is B: "I find the mood swings and the change from a calm person to being angry all the time hard to deal with." The frontal lobe of the brain controls affect, judgment and emotions. Dysfunction in this area results in findings such as emotional lability, changes in personality, inattentiveness, flat affect and inappropriate behavior. Question 18 Immediately following an acute battering incident in a violent relationship, the batterer may respond to the partner’s injuries by A) seeking medical help for the victim's injuries B) minimizing the episode and underestimating the victim’s injuries C) contacting a close friend and asking for help D) being very remorseful and assisting the victim with medical care Review Information: The correct answer is B: minimizing the episode and

underestimating the victim’s injuries Many batterers lack an understanding of the effects of their behavior on the victim and use excessive minimization and denial.

Question 19 The client who is receiving enteral nutrition through a gastrostomy tube has had 4 diarrhea stools in the past 24 hours. The nurse should A) review the medications the client is receiving B) increase the formula infusion rate C) increase the amount of water used to flush the tube D) attach a rectal bag to protect the skin

Review Information: The correct answer is A: review the medications the client is receiving Antibiotics and medications containing sorbitol may induce diarrhea. . Question 20 A postpartum mother is unwilling to allow the father to participate in the newborn's care, although he is interested in doing so. She states, "I am afraid the baby will be confused about who the mother is. Baby raising is for mothers, not fathers." The nurse's initial intervention should be what focus? A) Discuss with the mother sharing parenting responsibilities B) Set time aside to get the mother to express her feelings and concerns C) Arrange for the parents to attend infant care classes D) Talk with the father and help him accept the wife's decision Review Information: The correct answer is B: Set time aside to get the mother to express her feelings and concerns Non-judgmental support for expressed feelings may lead to resolution of competitive feelings in a new family. Cultural influences may also be clarified.

Question 21 The nurse is discussing nutritional requirements with the parents of an 18 month-old child. Which of these statements about milk consumption is correct?

A) May drink as much milk as desired B) Can have milk mixed with other foods C) Will benefit from fat-free cow's milk D) Should be limited to 3-4 cups of milk daily

Review Information: The correct answer is D: Should be limited to 3-4 cups of milk daily More than 32 ounces of milk a day considerably limits the intake of solid foods, resulting in a deficiency of dietary iron, as well as other nutrients. Question 22 Which of these parents’ comments about a newborn would most likely reveal an initial finding of a suspected pyloric stenosis? A) "I noticed a little lump a little above the belly button." B) "The baby seems hungry all the time." C) "Mild vomiting turned into vomiting that shot across the room." D) "We notice irritation and spitting up immediately after feedings." Review Information: The correct answer is C: "Mild vomiting turned into vomiting that shot across the room." Mild regurgitation or emesis that progresses to projectile vomiting is a pattern associated with pyloric stenosis as an initial finding. The other findings are present, though not immediately.

Question 23 The nurse is talking with a client. The client abruptly says to the nurse, "The moon is full. Astronauts walk on the moon. Walking is a good health habit." The client’s remarks most likely indicate A) neologisms B) flight of ideas C) loose associations D) word salad

Review Information: The correct answer is C: loose associations Though the client’s statements are not typical of logical communication, remarks 2 and 3 contain elements of the preceding sentence (moon, walk). Option A refers to making up words that have personal meaning to the client, and option B – flight of ideas defines nearly continuous flow of speech, jumping from one unconnected topic to another. Option D – word salad refers to stringing together real words into nonsense “sentences” that have no meaning for the listener. Question 24 The nurse is performing an assessment on a child with severe airway obstruction. Which finding would the nurse anticipate? A) Retractions in the intercostal tissues of the thorax B) Chest pain aggravated by respiratory movement C) Cyanosis and mottling of the skin D) Rapid, shallow respirations Review Information: The correct answer is A: Retractions in the intercostal tissues of the thorax Slight intercostal retractions are normal, however in disease states, especially in severe airway obstruction, retractions become extreme.

Question 25 A Hispanic client in the postpartum period refuses the hospital food because it is "cold." The best initial action by the nurse is to A) have the unlicensed assistive personnel (UAP) reheat the food if the client wishes B) ask the client what foods are acceptable or are unacceptable C) encourage her to eat for healing and strength D) schedule the dietitian to meet with the client as soon as possible

Review Information: The correct answer is B: ask the client what foods are acceptable or are unacceptable Many Hispanic women subscribe to the balance of hot and cold foods in the post partum period. What defines "cold" can best be explained by the client or family. Question 26 The nurse should recognize that physical dependence is accompanied by what

findings when alcohol consumption is first reduced or ended? A) Seizures B) Withdrawal C) Craving D) Marked tolerance Review Information: The correct answer is B: Withdrawal The early signs of alcohol withdrawal develop within a few hours after cessation or reduction of alcohol intake. Seizure activity is one withdrawal symptom but there are many others, like nausea and tremor.

Question 27 The nurse is preparing a 5 year-old for a scheduled tonsillectomy and adenoidectomy. The parents are anxious and concerned about the child's reaction to impending surgery. Which nursing intervention would best prepare the child? A) Introduce the child to all staff the day before surgery B) Explain the surgery 1 week prior to the procedure C) Arrange a tour of the operating and recovery rooms D) Encourage the child to bring a favorite toy to the hospital

Review Information: The correct answer is B: Explain the surgery 1 week prior to the procedure A 5 year-old can understand the surgery, and should be prepared well before the procedure. Most of these procedures are "same day" surgeries and do not require an overnight stay. Question 28 The nurse is monitoring the contractions of a woman in labor. A contraction is recorded as beginning at 10:00 A.M. and ending at 10:01 A.M. Another begins at 10:15 A.M. What is the frequency of the contractions? A) 14 minutes B) 10 minutes C) 15 minutes D) Nine minutes

Review Information: The correct answer is C: 15 minutes Frequency is the time from the beginning of one contraction to the beginning of the next contraction.

Question 29 The nurse is assigned to a client who has heart failure . During the morning rounds the nurse sees the client develop sudden anxiety, diaphoresis and dyspnea. The nurse auscultates, crackles bilaterally. Which nursing intervention should be performed first? A) Take the client's vital signs B) Place the client in a sitting position with legs dangling C) Contact the health care provider D) Administer the PRN antianxiety agent

Review Information: The correct answer is B: Place the client in a sitting position with legs dangling Place the client in a sitting position with legs dangling to pool the blood in the legs. This helps to diminish venous return to the heart and minimize the pulmonary edema. The result will enhance the client’s ability to breathe. The next actions would be to contact the heath care provider, then take the vital signs and then the administration of the antianxiety agent. Question 30 A client with emphysema visits the clinic. While teaching about proper nutrition, the nurse should emphasize that the client should A) eat foods high in sodium to increase sputum liquefaction B) use oxygen during meals to improve gas exchange C) perform exercise after respiratory therapy to enhance appetite D) cleanse the mouth of dried secretions to reduce risk of infection Review Information: The correct answer is B: use oxygen during meals to improve gas exchange Clients with emphysema breathe easier when using oxygen while eating.

Question 31 A home health nurse is caring for a client with a pressure sore that is red, with serous drainage, is 2 inches in diameter with loss of subcutaneous tissue. The appropriate dressing for this wound is A) transparent film dressing B) wet dressing with debridement granules C) wet to dry with hydrogen peroxide D) moist saline dressing Review Information: The correct answer is D: moist saline dressing This wound is a stage III pressure ulcer. The wound is red (granulation tissue) and does not require debridement. The wound must be protected for granulation tissue to proliferate. A moist dressing allows epithelial tissues to migrate more rapidly. Question 32 The father of an 8 month-old infant asks the nurse if his child's vocalizations are normal for his age. Which of the following would the nurse expect at this age? A) Cooing B) Imitation of sounds C) Throaty sounds D) Laughter Review Information: The correct answer is B: Imitation of sounds Imitation of sounds such as "da-da" is expected at this time.

Question 33 A mother asks about expected motor skills for a 3 year-old child. Which of the following would the nurse emphasize as normal at this age? A) Jumping rope B) Tying shoelaces C) Riding a tricycle D) Playing hopscotch

Review Information: The correct answer is C: Riding a tricycle Coordination is gained through large muscle use. A child of 3 has the ability to ride a tricycle. Question 34 An 18 month-old has been brought to the emergency room with irritability, lethargy over 2 days, dry skin, and increased pulse. Based upon the evaluation of these initial findings, the nurse would assess the child for additional findings of A) septicemia B) dehydration C) hypokalemia D) hypercalcemia

Review Information: The correct answer is B: dehydration Clinical findings of dehydration include lethargy, irritability, dry skin, and increased pulse.

Question 35 In taking the history of a pregnant woman, which of the following would the nurse recognize as the primary contraindication for breast feeding? A) Age 40 years B) Lactose intolerance C) Family history of breast cancer D) Use of cocaine on weekends Review Information: The correct answer is D: Use of cocaine on weekends Binge use of cocaine can be just as harmful to the breast fed newborn as regular use. Question 36 The school nurse suspects that a third grade child might have attention deficit hyperactivity disorder (ADHD). Prior to referring the child for further evaluation, the nurse should

A) observe the child's behavior on at least 2 occasions B) consult with the teacher about how to control impulsivity C) compile a history of behavior patterns and developmental accomplishments D) compare the child's behavior with classic signs and symptoms Review Information: The correct answer is C: compile a history of behavior patterns and developmental accomplishments A complete behavioral, and developmental history plays an important role in determining the diagnosis.

Question 37 In evaluating the growth of a 12 month-old child, which of these findings would the nurse expect to be present in the infant? A) Increased 10% in height B) 2 deciduous teeth C) Tripled the birth weight D) Head > chest circumference Review Information: The correct answer is C: Tripled the birth weight The infant usually triples his birth weight by the end of the first year of life. Height usually increases by 50% from birth length. A 12 month- old child should have approximately 6 teeth. ( estimate number of teeth by subtracting 6 from age in months, ie 12 – 6 = 6). By 12 months of age, head and chest circumferences are approximately equal. Question 38 A client who has been drinking for five years states that he drinks when he gets upset about "things" such as being unemployed or feeling like life is not leading anywhere. The nurse understands that the client is using alcohol as a way to deal with A) recreational and social needs B) feelings of anger C) life’s stressors D) issues of guilt and disappointment

Review Information: The correct answer is C: life’s stressors Alcohol is used by some people to manage anxiety and stress. The overall intent is to decrease negative feelings and increase positive feelings, but substance abuse itself eventually increases negative feelings.

Question 39 A client is receiving nitroprusside IV for the treatment of acute heart failure with pulmonary edema. What diagnostic lab value should the nurse monitor when a client is receiving this medication? A) Potassium level B) Arterial blood gasses C) Blood urea nitrogen D) Thiocyanate

Review Information: The correct answer is D: Thiocyanate Thiocyanate levels rise with the metabolism if nitroprusside is taken, and this can cause cyanide toxicity. Thiocyanate should not be over 1 millimole/liter. Question 40 A nurse is doing preconception counseling with a woman who is planning a pregnancy. Which of the following statements suggests that the client understands the connection between alcohol consumption and fetal alcohol syndrome? A) "I understand that a glass of wine with dinner is healthy." B) "Beer is not really hard alcohol, so I guess I can drink some." C) "If I drink, my baby may be harmed before I know I am pregnant." D) "Drinking with meals reduces the effects of alcohol." Review Information: The correct answer is C: "If I drink, my baby may be harmed before I know I am pregnant." Alcohol has the greatest teratogenic effect during organogenesis, in the first weeks of pregnancy. Therefore women considering a pregnancy should not drink.

When a client has left-sided weakness, what part of a sweater is put on first? (A) Both sleeves (B) Left sleeve (C) Client’s choice (D) Right sleeve Answer: (B) Left sleeve Exercises that move each muscle and joint are called: (A) adduction (B) range of motion (C) abduction (D) rotation Answer: (B) range of motion The Heimlich maneuver (abdominal thrust) is used for a client who has: (A) a blocked airway (B) a bloody nose (C) fallen out of bed (D) impaired eyesight Answer: (A) a blocked airway Which of the following is a correct measurement of urinary output? (A) 40 oz (B) 2 cups (C) 300 cc (D) 1 quart Answer: (C) 300 cc BEFORE taking the oral temperature of a client who has just finished a cold drink, the nurse aide should wait: (A) 10 to 20 minutes (B) 25 to 35 minutes (C) 45 to 55 minutes (D) at least 1 hour Answer: (A) 10 to 20 minutes These are just some sample testing questions for the CNA exam. We will be offering full sample CNA exams shortly that you will be able to download and study at your leisure.

1. Which of the following disorders is characterized by joint inflammation that is usually accompanied by pain and frequently accompanied by changes in structure? a. Synovitis b. Arthritis c. Bursitis d. Tendinitis 2. Which term refers to the expectoration of blood from the respiratory tract? a. A hemorrhage b. Hematopoiesis c. Hemoptysis d. Hemopexis 3. Which term describes lack of coordination in performing planned, purposeful movements, resulting from a neurologic deficit? a. Apraxia b. Ataxia c. Fasciculation d. Myokymia 4. An elevation in the partial pressure of carbon dioxide in arterial blood (PaCO2) indicates that the patient has: a. Hypernatremia b. Hypocalcemia c. Hypoxemia d. Hypercapnia 5. The latest laboratory values indicate that the patient has thrombocytopenia. The combining form penia means:

a. Rupture b. Deficiency c. Formation d. Stupor 6. A patient is admitted to the hospital with a urine specific gravity of 1.030, a temperature of 102°F (38.9° C), and flushed, dry skin. Based on these data, the nurse writes which of the following nursing diagnoses? a. Potential for impaired skin integrity b. Fluid volume deficit related to fever c. Potential for fluid volume deficit caused by fever d. Altered cardiopulmonary tissue perfusion related to fluid excess 7. The guidelines for writing an appropriate nursing diagnosis include all of the following except: a. State the diagnosis in terms of a problem, not a need b. Use nursing terminology to describe the patient’s response c. Use statements that assist in planning independent nursing interventions d. Use medical terminology to describe the probable cause of the patient’s response 8. Based on a physician’s order for oxygen by nasal catheter at 3 liters/ minute, an appropriate nursing order would be: a. Cover the tip of the catheter with a water-soluble lubricant before insertion. b. Measure the length of the catheter from the tip of the patient’s nose to the tip of the earlobe before insertion c. Add sterile distilled water to the humidification container, as needed d. All of the above

9. A nurse observes a dazed and apparently confused co-worker taking two diazepam (Valium) tablets by mouth as the co-worker is about to pour medications. What should the nurse do? a. Call the head nurse immediately before the co-worker pours and administers the medications b. Pour the medications for the co-worker while she goes for a cup of coffee c. Report the co-worker to hospital security because she may be addicted to drugs d. Watch the co-worker closely and report the incident to the head nurse at the end of the day. 10. A nurse manager notices that one of the staff nurses is always 15 to 20 minutes late. When the nurse manager discusses the problem with her, the nurse says that she has been late because her son’s nursery school does not open until 7 am. The nurse manager should respond by telling her to: a. Ask one of the night nurses to cover for her b. See if a neighbor can take the child to school c. Find out if other schools open earlier d. Find some way to solve the problem and be on time 11. A nurse has just moved to a new state, where she has accepted employment in a hospital-based hemodialysis unit. She needs information about her specific duties in caring for hemodialysis patients. She will find this information in: a. Policy statements set by the National Kidney Foundation b. The state’s nurse practice act c. Medicare and Medicaid regulations d. The hospital’s procedure manual 12. Which of the following is an example of nursing malpractice? a. The nurse administers penicillin to a patient with a documented history of allergy to the drug. The patient experiences an allergic reaction and has cerebral damage resulting from anoxia.

b. The nurse applies a hot water bottle or a heating pad to the abdomen of a patient with abdominal cramping c. The nurse assists a patient out of bed with the bed locked in position; the patient slips and fractures his right humerus d. The nurse administers the wrong medication to a patient and the patient vomits. This information is documented and reported to the physician and the nursing supervisor 13. Therapeutic communication is a significant aspect of patient care. Which of the following statements most clearly defines this concept? a. Therapeutic communication conveys feelings of warmth, acceptance, and empathy from the nurse to the patient in a nonjudgmental atmosphere b. Therapeutic communication is a reciprocal interaction based on trust and aimed at identifying patient needs and developing mutual goals c. Therapeutic communication is the assessment component of the nursing process, in which the nurse gathers health history information from the patient’s perspective d. Therapeutic communication is an interactional process in which the nurse purposefully reviews and assesses the conversation and its potential outcomes 14. Many factors can become barriers to communication. In which of the following situations would communication least likely be hindered? a. Mr. S., a 30-year-old Vietnamese immigrant, is admitted to the hospital with a fractured tibia; he speaks limited English b. Ms. M., age 58 and unmarried, is admitted to the hospital for breast surgery c. Mrs. R, age 26, is admitted to the hospital for a scheduled cesarean section; this is her first admission d. Mr. G., age 78, arrives at the hospital by ambulance after suffering a stroke at home 15. The assessment component of the nursing process requires effective communication to elicit a complete, relevant history from the patient and to identify patient problems. What role does communication play in the other areas of the nursing process? a. In the planning phase, effective therapeutic communication helps to establish nursing care priorities and patient-oriented goals

b. During the implementation phase, communication skills allow the nurse to assess the patient’s response to planned interventions c. During the evaluation phase, effective communication allows the nurse to find out from the patient if he is responding to treatment or if changes in treatment are necessary d. All of the above 16. All of the following would be considered objective assessment data for a patient admitted with diabetes mellitus except: a. + 2 urine glucose level; negative urine acetone level b. Chemstrip reading of 240 mg/dl c. Patient complaints of polydipsia d. Serum glucose level of 263 mg/dl 17. Which of the following statements about bowel sounds is accurate? a. Peristalsis causes bowel sounds b. Rapid, high-pitched, hyperactive bowel sounds indicate increased peristalsis c. Decreased bowel sounds can be a symptom of paralytic ileus d. All of the above 18. Independent nursing intervention commonly used for immobilized patients include all of the following except: a. Active or passive ROM exercises, body repositioning, and activities of daily living (ADLs) as tolerated b. Deep-breathing and coughing exercises with change of position every 2 hours c. Diaphragmatic and abdominal breathing exercises and increased hydration d. Weight bearing on a tilt table, total parenteral nutrition, and vitamin therapy 19. Independent nursing interventions commonly used for patients with pressure ulcers include: a. Changing the patient’s position regularly to minimize pressure

b. Applying a drying agent such as an antacid to decrease moisture at the ulcer site c. Debriding the ulcer to remove necrotic tissue, which can impede healing d. Placing the patient in a whirlpool bath containing povidone-iodine solution as tolerated 20. A female patient has gained 24 lb after being admitted to the hospital. “I’m such a horse; I just can’t stand myself like this,” she tells the nurse. After assessing the patient, the nurse writes the following nursing diagnosis: Body image disturbance. To arrive at this diagnosis, the nurse should include which of the following assessment findings? a. The patient’s perception of her body before the hospitalization and weight gain b. The significance the patient places on these changes c. The patient’s feelings about her body d. All of the above 21. Stressors cause the release of the mineralocorticoid aldosterone, which regulates sodium absorption and potassium excretion in the renal tubules, resulting in: a. The need for supplemental potassium b. The need for a low-sodium (500-mg) diet c. The conservation of water and maintenance of blood volume d. Increased diuresis 22. In planning the care of a patient who is exposed to multiple stressors such as separation from loved ones, anxiety about impending surgery, and concern about potential complications or death, the nurse must: a. Use both a structured and an unstructured format when interviewing the patient b. Know the stressors affecting the patient c. Develop the expected outcomes for each nursing diagnosis written for this patient d. All of the above

23. An accurate method of calculating the daily urine output of an incontinent patient wearing pads or diapers is to: a. Estimate the urine output b. Count the number of urine saturated pads c. Weigh a dry pad and each urine saturated pad and use a conversion calibration to calculate the urine output d. Weigh all the urine-saturated pads together and use a conversion calibration to calculate the urine output 24. A fashion model is admitted via the emergency room with facial and chest burns. Her hospital stay includes 10 days in the intensive care unit and 5 days on the regular hospital unit. The patient has not been eating or sleeping and refuses to perform her activities of daily living (ADLs). She refuses to work with speech and physical therapists. Which of the following nursing diagnoses might appears on the patient’s current care plan? a. Potential for noncompliance: Self-harm related to disturbed body image b. Self-care deficit related to knowledge deficit and disturbed body image c. Disturbance in self-concept: Personal identifying related to self-esteem d. Disturbance in self-concept related to altered thought process 25. White the nurse is providing a patient’s personal hygiene, she observes that his skin is excessively dry. During this procedure the patient tells her that he is very thirsty. An appropriate nursing diagnosis would be: a. Potential for impaired skin integrity related to altered gland function b. Potential for impaired skin integrity related to dehydration c. Impaired skin integrity relate to dehydration d. Impaired skin integrity related to altered circulation

View Questions 1. Answer – B. Arthritis is characterized by joint inflammation that is usually accompanied by pain and frequently accompanied by changes in structure. Synovitis is the inflammation of the synovial membrane, typically resulting from a traumatic injury or an aseptic wound. Bursitis is the inflammation of a bursa, typically one located between a bony prominence and a muscle or tendon. Tendinitis is the inflammation of tendon. 2. Answer – C. Hemoptysis is the expectoration of blood from the respiratory tract. A hemorrhage is abnormal internal or external bleeding. Hematopoiesis is blood cell formation. Hemopexis is blood coagulation. 3. Answer – B. Ataxia is lack of coordination in performing planned, purposeful movements, typically resulting from a neurologic deficit. Apraxia is the inability to perform purposeful movements even though no neuromuscular deficit exists. Fasciculations are fine twitching movements. Myokymia is a transient, spontaneous movement that occurs in muscle groups after strenuous exercise. 4. Answer – D. Hypercapnia is an elevation in the partial pressure of carbon dioxide in arterial blood (PaCO2). Hypernatremia is an elevated level of sodium in venous blood (more than 145 mEq/liter). Hypocalcemia is a decreased level of calcium in venous blood (less than 9 mg/dl). Hypoxemia is a reduced level of oxygen in arterial blood (less than 80 mm Hg while breathing room air). 5. Answer – B. The combining form penia means deficiency, as in thrombocytopenia (deficiency in the number of circulating blood plates). Rrhexis is a combining form meaning rupture, as in enterorrhexis (rupture of the intestine). Plast is a combining form meaning formation, as in rhino-plasty (formation of a nose using plastic surgery). Narco is a combining form meaning stupor, as in narcolepsy (a condition marked by recurrent attacks of drowsiness and sleep). 6. Answer – B. Fluid volume deficit related to fever is the appropriate nursing diagnosis based on this assessment. Potential for impaired skin integrity states a possible patient response. Potential for fluid volume deficit caused by fever implies a cause-and-effect relationship, which a nursing diagnosis should never do. Altered cardiopulmonary tissue perfusion related to fluid excess is an incorrect diagnosis based on a misinterpretation of the data. 7. Answer – D. A nursing diagnosis is a statement about a patient’s actual or potential health problem that is within the scope of independent nursing intervention. Medical terminology is never a part of the nursing diagnosis. An appropriate nursing diagnosis

would be ineffective breathing pattern related to chest pain rather than ineffective breathing pattern caused by angina. 8. Answer – D. A water-soluble lubricant must be applied to the tip of the catheter to decrease friction and the risk of injury to the patient’s nasal mucosa. (If petrolatum or mineral oil were applied to the catheter and then aspirated, the patient could develop a lipoid pneumonia) The distance from the tip of the nose to the tip of the earlobe is the approximate distance from the point of insertion to the oropharynx. Sterile distilled water must be used to humidity the oxygen because oxygen administered by itself is a dry gas that can irritate the mucosa. 9. Answer – A. Patient safety is the major concern in this situation. According to the International Council of Nurses’ Code for Nurses: “The nurse [should] take appropriate action to safeguard the individual when his or her care is endangered by a co-worker or any other person.” In this case, talking with the head nurse immediately would be the best way to safeguard the patient’s safety. The nurse isn’t necessarily an addict, she may be abusing a prescription medication. 10. Answer – D. It is the staff nurse’s responsibility to be on time. The nurse manager should not assume a responsibility that belongs to the nurse. 11. Answer – D. Although Medicare and Medicaid regulations and suggestions made by such groups as the National Kidney Foundation may serve as guidelines, a hospital’s procedure manual details how the nurse should perform her specific duties. A state’s nurse practice act defines the scope of practice within that state, but not the specifics for each area of practice. 12. Answer – A. The three elements necessary to establishes nursing malpractice are nursing error (administering penicillin to a patient with a documented allergy to the drug), injury (cerebral damage), and proximal cause (administering the penicillin caused the cerebral damage). Applying a hot water bottle or healing pad to a patient without a physician’s order does not include the three required components. Assisting a patient out of bed with the bed locked in position is the correct nursing practice; therefore, the fracture was not the result of malpractice. Administering an incorrect medication is a nursing error; however, if such action resulted in a serious illness or chronic problem, the nurse could be sued for malpractice. 13. Answer – B. Therapeutic communication is a two way, deliberative interaction between the patient and nurse in which they establish mutually acceptable, achievable goals of care. Before the patient can feel comfortable discussing his problems, however, and atmosphere of trust and acceptance must be established. 14. Answer – C. Many variables affect patient nurse communication, including the patient’s cultural beliefs, experiences with hospitalization, age, emotional needs, and problems with speech, hearing, or comprehension. A patient admitted to the hospital for the first time for a scheduled cesarean section is probably anxious, but she had

time to plan for the procedure, does not bring negative experiences from previous hospitalizations, and in most cases looks forward to the birth. 15. Answer – D. Therapeutic communication is a fundamental component at all phases of the nursing process. In the planning phase, it allows the patient and nurse to formulate mutually acceptable and patient-oriented goals, which are the basis for developing an individualized care plan. In the implementation phase, effective communication is necessary for teaching the patient, motivating him to achieve goals, and assessing patient outcomes. Finally, in the evaluation phase, it is required to determine how well the patient has responded to interventions. 16. Answer – C. Objective data are those which can be measured, like glucose levels. A complaint of polydipsia is subjective information obtained from the patient. 17. Answer – D. Peristalsis is the muscular, rhythmic movement in the bowel wall that pushes food along the digestive tract distally. Increased bowel motility is indicated by rapid, high-pitched, hyperactive bowel sounds. Decreased bowel sounds, caused by decreased bowel motility, can be the initial sign of paralytic ileus (adynamic intestinal obstruction resulting from the lack of peristalsis), a common occurrence following abdominal surgery. 18. Answer – D. The use of a tilt table for weight-beating exercises, parenteral nutrition, and vitamin therapy are not independent nursing interventions because they require a physician’s order. Unless specifically contraindicated, the independent nursing interventions listed in A, B, and C may be part of the nursing care plan for an immobilized patient. 19. Answer – A. Independent nursing interventions for a patient with pressure ulcers commonly include changing his position several times each day to avoid pressure to any part of his body, especially the involved area. Drying agents, which are prescribed by a physician, are contraindicated because wounds need moisture to heal. Whirlpool therapy and chemical debridement must be prescribed, and surgical debridement is done by the physician. 20. Answer – D. All of the choices will help the nurse determine the extent of the problem. For example, asking how the patient felt about her body before hospitalization will help the nurse determine whether the disturbed body image is a crisis brought on by the weight gain or a long-standing problem. Asking what the change means to her will reveal whether she feels she has control over what is happening or believes the change is permanent. Body image is also related to how we think we compare to others or whether others find us attractive. 21. Answer – C. Because aldosterone regulates the body’s sodium and potassium levels, it acts as an adaptive mechanism in maintaining blood volume and conserving water. Supplemental potassium usually is given to a patient with a low serum potassium level or one who is receiving a diuretic or other medication – such as digoxin – that

has a mild diuretic effect. A low-sodium diet is usually prescribed for a patient with a high serum sodium level, as in congestive heart failure (CHF), hypertension, or prolonged episodes of edema. Diuresis is increased naturally when a healthy patient increases his intake of fluids, especially those containing caffeine. Patients receiving diuretics also experience increased diuresis. 22. Answer – D. Interviewing the patient – in both a structured and an unstructured format – is an important part of the initial nursing assessment. The structured format uses questions that require a yea-or-no answer to help the nurse obtain information; the unstructured format uses open-ended questions that allow the patient to express himself more fully. The interview helps the nurse and patient identify the stressors and develop appropriate outcomes. 23. Answer – C. Calculating the difference in weight between a dry pad and a urine saturated pad using conversion calibration will provide an accurate measure of urine output. For example, if the difference between the dry pad and the urine-saturated pad is 200 g, the urine output would be 200 ml (1g = 1 ml). The other methods will provide only an estimate of urine output. 24. Answer – C. Disturbances in self-concept may manifest themselves as signs and symptoms of depression, such as changes in sleep patterns, eating habits, and energy levels. The other nursing diagnoses are not supported by the given situation. 25. Answer – C. An appropriate nursing diagnosis for a patient with excessively dry skin is Impaired skin integrity (actual not potential) – in this case, related to dehydration because the patient complains of thirst. Altered circulation is not usually an etiologic factor for dry skin.

1. Which intervention is an example of primary prevention? a. Administering digoxin (Lanoxicaps) to a patient with heart failure b. Administering a measles, mumps, and rubella immunization to an infant c. Obtaining a Papanicolaou smear to screen for cervical cancer d. Using occupational therapy to help a patient cope with arthritis 2. The nurse in charge is assessing a patient’s abdomen. Which examination technique should the nurse use first? a. Auscultation b. Inspection c. Percussion d. Palpation 3. Which statement regarding heart sounds is correct? a. S1 and S2 sound equally loud over the entire cardiac area. b. S1 and S2 sound fainter at the apex c. S1 and S2 sound fainter at the base d. S1 is loudest at the apex, and S2 is loudest at the base 4. The nurse in charge identifies a patient’s responses to actual or potential health problems during which step of the nursing process? a. Assessment b. Nursing diagnosis c. Planning d. Evaluation 5. A female patient is receiving furosemide (Lasix), 40 mg P.O. b.i.d. in the plan of care, the nurse should emphasize teaching the patient about the importance of consuming: a. Fresh, green vegetables b. Bananas and oranges c. Lean red meat d. Creamed corn 6. The nurse in charge must monitor a patient receiving chloramphenicol for adverse drug reaction. What is the most toxic reaction to chloramphenicol? a. Lethal arrhythmias b. Malignant hypertension c. Status epilepticus d. Bone marrow suppression 7. A female patient is diagnosed with deep-vein thrombosis. Which nursing diagnosis should receive highest priority at this time? a. Impaired gas exchanges related to increased blood flow b. Fluid volume excess related to peripheral vascular disease c. Risk for injury related to edema d. Altered peripheral tissue perfusion related to venous congestion

8. When positioned properly, the tip of a central venous catheter should lie in the: a. Superior vena cava b. Basilica vein c. Jugular vein d. Subclavian vein 9. Nurse Margareth is revising a client’s care plan. During which step of the nursing process does such revision take place? a. Assessment b. Planning c. Implementation d. Evaluation 10. A 65-year-old female who has diabetes mellitus and has sustained a large laceration on her left wrist asks the nurse, “How long will it take for my scars to disappear?” which statement would be the nurse’s best response? a. “The contraction phase of wound healing can take 2 to 3 years.” b. “Wound healing is very individual but within 4 months the scar should fade.” c. “With your history and the type of location of the injury, it’s hard to say.” d. “If you don’t develop an infection, the wound should heal any time between 1 and 3 years from now.” 11. a. b. c. d. One aspect of implementation related to drug therapy is: Developing a content outline Documenting drugs given Establishing outcome criteria Setting realistic client goals

12. A female client is readmitted to the facility with a warm, tender, reddened area on her right calf. Which contributing factor would the nurse recognize as most important? a. A history of increased aspirin use b. Recent pelvic surgery c. An active daily walking program d. A history of diabetes 13. Which intervention should the nurse in charge try first for a client that exhibits signs of sleep disturbance? a. Administer sleeping medication before bedtime b. Ask the client each morning to describe the quantity of sleep during the previous night c. Teach the client relaxation techniques, such as guided imagery, medication, and progressive muscle relaxation d. Provide the client with normal sleep aids, such as pillows, back rubs, and snacks 14. While examining a client’s leg, the nurse notes an open ulceration with visible granulation tissue in the wound. Until a wound specialist can be contacted, which type of

dressings is most appropriate for the nurse in charge to apply? a. Dry sterile dressing b. Sterile petroleum gauze c. Moist, sterile saline gauze d. Povidone-iodine-soaked gauze 15. A male client in a behavioral-health facility receives a 30-minute psychotherapy session, and provider uses a current procedure terminology (CPT) code that bills for a 50minute session. Under the False Claims Act, such illegal behavior is known as: a. Unbundling b. Overbilling c. Upcoding d. Misrepresentation 16. A nurse assigned to care for a postoperative male client who has diabetes mellitus. During the assessment interview, the client reports that he’s impotent and says that he’s concerned about its effect on his marriage. In planning this client’s care, the most appropriate intervention would be to: a. Encourage the client to ask questions about personal sexuality b. Provide time for privacy c. Provide support for the spouse or significant other d. Suggest referral to a sex counselor or other appropriate professional 17. Using Abraham Maslow’s hierarchy of human needs, a nurse assigns highest priority to which client need? a. Security b. Elimination c. Safety d. Belonging 18. A male client is on prolonged bed rest has developed a pressure ulcer. The wound shows no signs of healing even though the client has received skin care and has been turned every 2 hours. Which factor is most likely responsible for the failure to heal? a. Inadequate vitamin D intake b. Inadequate protein intake c. Inadequate massaging of the affected area d. Low calcium level 19. A female client who received general anesthesia returns from surgery. Postoperatively, which nursing diagnosis takes highest priority for this client? a. Acute pain related to surgery b. Deficient fluid volume related to blood and fluid loss from surgery c. Impaired physical mobility related to surgery d. Risk for aspiration related to anesthesia

20. Nurse Cay inspects a client’s back and notices small hemorrhagic spots. The nurse documents that the client has: a. Extravasation b. Osteomalacia c. Petechiae d. Uremia 21. Which document addresses the client’s right to information, informed consent, and treatment refusal? a. Standard of Nursing Practice b. Patient’s Bill of Rights c. Nurse Practice Act d. Code for Nurses 22. If a blood pressure cuff is too small for a client, blood pressure readings taken with such a cuff may do which of the following? a. Fail to show changes in blood pressure b. Produce a false-high measurement c. Cause sciatic nerve damage d. Produce a false-low measurement 23. Nurse Danny has been teaching a client about a high-protein diet. The teaching is successful if the client identifies which meal as high in protein? a. Baked beans, hamburger, and milk b. Spaghetti with cream sauce, broccoli, and tea c. Bouillon, spinach, and soda d. Chicken cutlet, spinach, and soda 24. A male client is admitted to the hospital with blunt chest trauma after a motor vehicle accident. The first nursing priority for this client would be to: a. Assess the client’s airway b. Provide pain relief c. Encourage deep breathing and coughing d. Splint the chest wall with a pillow 25. A newly hired charge nurse assesses the staff nurses as competent individually but ineffective and nonproductive as a team. In addressing her concern, the charge nurse should understand that the usual reason for such a situation is: a. Unhappiness about the charge in leadership b. Unexpected feeling and emotions among the staff c. Fatigue from overwork and understaffing d. Failure to incorporate staff in decision making 26. A male client blood test results are as follows: white blood cell (WBC) count, 100ul; hemoglobin (Hb) level, 14 g/dl; hematocrit (HCT), 40%. Which goal would be most important for this client?

a. Promote fluid balance b. Prevent infection c. Promote rest d. Prevent injury 27. Following a tonsillectomy, a female client returns to the medical-surgical unit. The client is lethargic and reports having a sore throat. Which position would be most therapeutic for this client? a. Semi-Fowler’s b. Supine c. High-Fowler’s d. Side-lying 28. Nurse Berri inspects a client’s pupil size and determines that it’s 2 mm in the left eye and 3 mm in the right eye. Unequal pupils are known as: a. Anisocoria b. Ataxia c. Cataract d. Diplopia 29. The nurse in charge is caring for an Italian client. He’s complaining of pain, but he falls asleep right after his complaint and before the nurse can assess his pain. The nurse concludes that: a. He may have a low threshold for pain b. He was faking pain c. Someone else gave him medication d. The pain went away 30. A female client is admitted to the emergency department with complaints of chest pain shortness of breath. The nurse’s assessment reveals jugular vein distention. The nurse knows that when a client has jugular vein distension, it’s typically due to: a. A neck tumor b. An electrolyte imbalance c. Dehydration d. Fluid overload

1. Answer B. Immunizing an infant is an example of primary prevention, which aims to prevent health problems. Administering digoxin to treat heart failure and obtaining a smear for a screening test are examples for secondary prevention, which promotes early detection and treatment of disease. Using occupational therapy to help a patient cope with arthritis is an example of tertiary prevention, which aims to help a patient deal with the residual consequences of a problem or to prevent the problem from recurring. 2. Answer B. Inspection always comes first when performing a physical examination. Percussion and palpation of the abdomen may affect bowel motility and therefore should follow auscultation. 3. Answer D. The S1 sound—the “lub” sound—is loudest at the apex of the heart. It sounds longer, lower, and louder there than the S2 sounds. The S2—the “dub” sound—is loudest at the base. It sounds shorter, sharper, higher, and louder there than S1. 4. Answer B. The nurse identifies human responses to actual or potential health problems during the nursing diagnosis step of the nursing process. During the assessment step, the nurse systematically collects data about the patient or family. During the planning step, the nurse develops strategies to resolve or decrease the patient’s problem. During the evaluation step, the nurse determines the effectiveness of the plan of care. 5. Answer B. Because furosemide is a potassium-wasting diuretic, the nurse should plan to teach the patient to increase intake of potassium-rich foods, such as bananas and oranges. Fresh, green vegetables; lean red meat; and creamed corn are not good sources of potassium. 6. Answer D. The most toxic reaction to chloramphenicol is bone marrow suppression. Chloramphenicol is not known to cause lethal arrhythmias, malignant hypertension, or status epilepticus. 7. Answer D. Altered peripheral tissue perfusion related to venous congestion” takes highest priority because venous inflammation and clot formation impede blood flow in a patient with deep-vein thrombosis. Option A is incorrect because impaired gas exchange is related to decreased, not increased, blood flow. Option B is inappropriate because no evidence suggest that this patient has a fluid volume excess. Option C may be warranted but is secondary to altered tissue perfusion. 8. Answer A. When the central venous catheter is positioned correctly, its tip lies in the superior vena cava, inferior vena cava, or the right atrium—that is, in central venous circulation. Blood flows unimpeded around the tip, allowing the rapid infusion of large amounts of fluid directly into circulation. The basilica, jugular, and subclavian veins are common insertion sites for central venous catheters. 9. Answer D. During the evaluation step of the nursing process the nurse determines whether the goals established in the care plan have been achieved, and evaluates the success of the plan. If a goal is unmet or partially met the nurse reexamines the data and revises the plan.

Assessment involves data collection. Planning involves setting priorities, establishing goals, and selecting appropriate interventions. 10. Answer C. Wound healing in a client with diabetes will be delayed. Providing the client with a time frame could give the client false information. 11. Answer B. Although documentation isn’t a step in the nursing process, the nurse is legally required to document activities related to drug therapy, including the time of administration, the quantity, and the client’s reaction. Developing a content outline, establishing outcome criteria, and setting realistic client goals are part of planning rather than implementation. 12. Answer B. The client shows signs of deep vein thrombosis (DVT). The pelvic area is rich in blood supply, and thrombophlebitis of the deep vein is associated with pelvic surgery. Aspirin, an antiplatelet agent, and an active walking program help decrease the client’s risk of DVT. In general, diabetes is a contributing factor associated with peripheral vascular disease. 13. Answer D. The nurse should begin with the simplest interventions, such as pillows or snacks, before interventions that require greater skill such as relaxation techniques. Sleep medication should be avoided whenever possible. At some point, the nurse should do a thorough sleep assessment, especially if common sense interventions fail. 14. Answer C. Moist, sterile saline dressings support would heal and are costeffective. Dry sterile dressings adhere to the wound and debride the tissue when removed. Petroleum supports healing but is expensive. Povidoneiodine can irritate epithelial cells, so it shouldn’t be left on an open wound. 15. Answer C. Upcoding is the practice of using a CPT code that’s reimbursed at a higher rate than the code for the service actually provided. Unbundling, overbilling, and misrepresentation aren’t the terms used for this illegal practice. 16. Answer D. The nurse should refer this client to a sex counselor or other professional. Making appropriate referrals is a valid part of planning the client’s care. The nurse doesn’t normally provide sex counseling. Therefore, providing time for privacy and providing support for the spouse or significant other are important, but not as important as referring the client to a sex counselor. 17. Answer B. According to Maslow, elimination is a first-level or physiological need, and therefore takes priority over all other needs. Security and safety are second-level needs; belonging is a third-level need. Second- and thirdlevel needs can be met only after a client’s first-level needs have been satisfied. 18. Answer B. A client on bed rest suffers from a lack of movement and a negative nitrogen balance. Therefore, inadequate protein intake impairs wound healing. Inadequate vitamin D intake and low calcium levels aren’t factors in poor healing for this client. A pressure ulcer should never be massaged.

19. Answer D. Risk for aspiration related to anesthesia takes priority for thins client because general anesthesia may impair the gag and swallowing reflexes, possibly leading to aspiration. The other options, although important, are secondary. 20. Answer C. Petechiae are small hemorrhagic spots. Extravasation is the leakage of fluid in the interstitial space. Osteomalacia is the softening of bone tissue. Uremia is an excess of urea and other nitrogen products in the blood. 21. Answer B. The Patient’s Bill of Rights addresses the client’s right to information, informed consent, timely responses to requests for services, and treatment refusal. A legal document, it serves as a guideline for the nurse’s decision making. Standards of Nursing Practice, the Nurse Practice Act, and the Code for Nurses contain nursing practice parameters and primarily describe the use of the nursing process in providing care. 22. Answer B. Using an undersized blood pressure cuff produces a falsely elevated blood pressure because the cuff can’t record brachial artery measurements unless it’s excessively inflated. The sciatic nerve wouldn’t be damaged by hyperinflation of the blood pressure cuff because the sciatic nerve is located in the lower extremity. 23. Answer A. Baked beans, hamburger, and milk are all excellent sources of protein. The spaghetti-broccoli-tea choice is high in carbohydrates. The bouillon-spinach-soda choice provides liquid and sodium as well as some iron, vitamins, and carbohydrates. Chicken provides protein but the chicken-spinach-soda combination provides less protein than the baked beans-hamburger-milk selection. 24. Answer A. The first priority is to evaluate airway patency before assessing for signs of obstruction, sternal retraction, stridor, or wheezing. Airway management is always the nurse’s first priority. Pain management and splinting are important for the client’s comfort, but would come after airway assessment. Coughing and deep breathing may be contraindicated if the client has internal bleeding and other injuries. 25. Answer B. The usual or most prevalent reason for lack of productivity in a group of competent nurses is inadequate communication or a situation in which the nurses have unexpected feeling and emotions. Although the other options could be contributing to the problematic situation, they’re less likely to be the cause. 26. Answer B. The client is at risk for infection because WBC count is dangerously low. Hb level and HCT are within normal limits; therefore, fluid balance, rest, and prevention of injury are inappropriate. 27. Answer D. Because of lethargy, the post tonsillectomy client is at risk for aspirating blood from the surgical wound. Therefore, placing the client in the side-lying position until he awake is best. The semi-Fowler’s, supine, and high-Fowler’s position don’t allow for adequate oral drainage in a lethargic post tonsillectomy client, and increase the risk of blood aspiration.

28. Answer A. Unequal pupils are called anisocoria. Ataxia is uncoordinated actions of involuntary muscle use. A cataract is an opacity of the eye’s lens. Diplopia is double vision. 29. Answer A. People of Italian heritage tend to verbalize discomfort and pain. The pain was real to the client, and he may need medication when he wakes up. 30. Answer D. Fluid overload causes the volume of blood within the vascular system to increase. This increase causes the vein to distend, which can be seen most obviously in the neck veins. A neck tumor doesn’t typically cause jugular vein distention. An electrolyte imbalance may result in fluid overload, but it doesn’t directly contribute to jugular vein distention.

1. A 65-year-old patient with pneumonia is receiving garamycin (Gentamicin). It would be MOST important for a nurse to monitor which of the following laboratory values in this patient? (A) Hemoglobin and hematocrit. (B) BUN and creatinine. (C) Platelet count and clotting time. (D) Sodium and potassium. 2. A 22-year-old man is admitted to the hospital with complaints of fatigue and weight loss. Physical examination reveals pallor and multiple bruises on his arms and legs. The results of the patients tests reveal acute lymphocytic leukemia and thrombocytopenia. Which of the following nursing diagnoses MOST accurately reflects his condition? (A) Potential for injury. (B) Self-care deficit. (C) Potential for self-harm. (D) Alteration in comfort.

3. To enhance the percutaneous absorption of nitroglycerine ointment, it would be MOST important for the nurse to select a site that is (A) muscular. (B) near the heart. (C) non-hairy. (D) over a bony prominence. 4. A man is admitted to the Telemetry Unit for evaluation of complaints of chest pain. Eight hours after admission, the patient goes into ventricular fibrillation. The physician defibrillates the patient. The nurse understands that the purpose of defibrillation is to: (A) increase cardiac contractility and cardiac output. (B) cause asystole so the normal pacemaker can recapture. (C) reduce cardiac ischemia and acidosis. (D) provide energy for depleted myocardial cells. 5. A patient is to receive 3,000 ml of 0.9% NaCl IV in 24 hours. The intravenous set delivers 15 drops per milliliter. The nurse should regulate the flow rate so that the patient receives how many drops of fluid per minute? (A) 21 (B) 28 (C) 31 (D) 42

ANSWERS AND RATIONALE

1. The correct answer is B.
Question: Which lab values should you monitor for a patient receiving Gentamicin? Needed Info: Gentamicin: broad spectrum antibiotic. Side effects: neuromuscular blockage, ototoxic to eighth cranial nerve (tinnitus, vertigo, ataxia, nystagmus, hearing loss), nephrotoxic. Nursing responsibilities: monitor renal function, force fluids, monitor hearing acuity. Draw blood for peak levels 1 hr. after IM and 30 min – 1 hr. after IV infusion, draw blood for trough just before next dose. (A) Hemoglobin and hematocrit — can cause anemia; less common (B) BUN and creatinine — CORRECT: nephrotoxic; will see proteinuria, oliguria, hematuria, thirst, increased BUN, decreased creatine clearance (C) Platelet count and clotting time — do not usually change (D) Sodium and potassium — hypokalemia infrequent problem

2. The correct answer is A.
Question: What nursing diagnosis is seen with acute lymphocytic leukemia and thromocytopenia? Needed Info: Thromocytopenia: decreased platelet count increases the patient’s risk for injury, normal count: 200,000-400,000 per mm3. Leukemia: group of malignant disorders involving overproduction of immature leukocytes in bone marrow. This shuts down normal bone marrow production of erythrocytes, platelets, normal leukocytes. Causes anemia, leukopenia, and thrombocytopenia leading to infection and hemorrhage. Symptoms: pallor of nail beds and conjunctiva, petechiae (small hemorrhagic spot on skin), tachycardia, dyspnea, weight loss, fatigue. Treatment: chemotherapy, antibiotics, blood transfusions, bone marrow transplantation. Nursing responsibilities: private room, no raw fruits or vegs, small frequent meals, O2, good skin care. (A) Potential for injury — CORRECT: low platelet increases risk of bleeding from even minor injuries. Safety measures: shave with an electric razor, use soft tooth brush, avoid SQ or IM meds and invasive procedures (urinary drainage catheter or a nasogastric tube), side-rails up, remove sharp objects, frequently assess for signs of bleeding, bruising, hemorrhage. (B) Self-care deficit — may feel weak, doesn’t address condition (C) Potential for self-harm — implies risk for purposeful self-injury, not given any info, assumption (D) Alteration in comfort — patient is not comfortable, and comfort measures would address problem

3. The correct answer is C.
Question: What is the best site for nitroglycerine ointment? Strategy: Think about each site. Needed Info: Nitroglycerine: used in treatment of angina pectoris to reduce ischemia and relieve pain by decreasing myocardial oxygen consumption; dilates veins and arteries. Side effects: throbbing headache, flushing, hypotension, tachycardia. Nursing responsibilities: teach appropriate administration, storage, expected pain relief, side effects. Ointment applied to skin; sites rotated to avoid skin irritaion. Prolonged effect up to 24 hours. (A) muscular — not most important (B) near the heart — not most important (C) non-hairy — CORRECT: skin site free of hair will increase absorption; avoid distal part of extremities due to less than maximal absorption (D) over a bony prominence — most important is that the site be non-hairy

4. The correct answer is B.
Question: Why is a patient defibrillated? Strategy: Think about each answer choice. Needed Info: Defibrillation: produces asystole of heart to provide opportunity for natural pacemaker (SA node) to resume as pacer of heart activity. (A) increase cardiac contractility and cardiac output — inaccurate (B) cause asystole so the normal pacemaker can recapture — CORRECT: allows SA node to resume as pacer of heart activity (C) reduce cardiac ischemia and acidosis — inaccurate (D) provide energy for depleted myocardial cells — inaccurate

5. The correct answer is C.
Question: How should you regulate the IV flow rate? Strategy: Use formula and avoid making math errors. Needed Info: total volume x the drop factor divided by the total time in minutes. (A) 21 — inaccurate (B) 28 — inaccurate (C) 31 — CORRECT: 3,000 x 15 divided by 24 x 60 (D) 42 — inaccurate

Psychosocial Integrity 1. An adolescent male being treated for depression arrives with his family at the Adolescent Day Treatment Center for an initial therapy meeting with the staff. The nurse explains that one of the goals of the family meeting is to encourage the adolescent to: (A) trust the nurse who will solve his problem. (B) learn to live with anxiety and tension. (C) accept responsibility for his actions and choices. (D) use the members of the therapeutic milieu to solve his problems. 2. A 23-year-old-woman comes to the emergency room stating that she had been raped. Which of the following statements BEST describes the nurse’s responsibility concerning written consent?

(A) The nurse should explain the procedure to the patient and ask her to sign the consent form. (B) The nurse should verify that the consent form has been signed by the patient and that it is attached to her chart. (C) The nurse should tell the physician that the patient agrees to have the examination. (D) The nurse should verify that the patient or a family member has signed the consent form. 3. The nurse cares for an elderly patient with moderate hearing loss. The nurse should teach the patient’s family to use which of the following approaches when speaking to the patient? (A) Raise your voice until the patient is able to hear you. (B) Face the patient and speak quickly using a high voice. (C) Face the patient and speak slowly using a slightly lowered voice. (D) Use facial expressions and speak as you would normally. 4. A 52-year-old man is admitted to a hospital after sustaining a severe head injury in an automobile accident. When the patient dies, the nurse observes the patient’s wife comforting other family members. Which of the following interpretations of this behavior is MOST justifiable? (A) She has already moved through the stages of the grieving process. (B) She is repressing anger related to her husband’s death. (C) She is experiencing shock and disbelief related to her husband’s death. (D) She is demonstrating resolution of her husband’s death. 5. After two weeks of receiving lithium therapy, a patient in the psychiatric unit becomes depressed. Which of the following evaluations of the patient’s behavior by the nurse would be MOST accurate? (A) The treatment plan is not effective; the patient requires a larger dose of lithium. (B) This is a normal response to lithium therapy; the patient should continue with the current treatment plan. (C) This is a normal response to lithium therapy; the patient should be monitored for suicidal behavior. (D) The treatment plan is not effective; the patient requires an antidepressant.

ANSWERS AND RATIONALE

1. The correct answer is C.
Question: What is the goal of family therapy?

Needed Info: Symptoms of depression: a low self-esteem, obsessive thoughts, regressive behavior, unkempt appearance, a lack of energy, weight loss, decreased concentration, withdrawn behavior. (A) trust the nurse who will solve his problem — not realistic (B) learn to live with anxiety and tension — minimizes concerns (C) accept responsibility for his actions and choices — CORRECT (D) use the members of the therapeutic milieu to solve his problems — must do it himself

2. The correct answer is B.
Question: What is your responsibility concerning informed consent? Needed Info: Physician’s responsibility to obtain informed consent. (A) The nurse should explain the procedure to the patient and ask her to sign the consent form — Physician should get patient to sign consent (B) The nurse should verify that the consent form has been signed by the patient and that it is attached to her chart — CORRECT (C) The nurse should tell the physician that the patient agrees to have the examination — Physician should explain procedure and get consent form signed (D) The nurse should verify that the patient or a family member has signed the consent form — must be signed by patient unless unable to do

3. The correct answer is C.
Question: What should you do to communicate with a person with a moderate hearing loss? Needed Info: Presbycusis: age-related hearing loss due to inner ear changes. Decreased ability to hear high sounds. (A) Raise your voice until the patient is able to hear you — would result in high tones patient unable to hear (B) Face the patient and speak quickly using a high voice — usually unable to hear high tones (C) Face the patient and speak slowly using a slightly lowered voice — CORRECT: also decrease background noise; speak at a slow pace, use nonverbal cues (D) Use facial expressions and speak as you would normally — nonverbal cues help, but need low tones

4. The correct answer is C.
Question: What is the reason for the wife’s behavior?

Needed Info: Stages of grief: 1) shock and disbelief, 2) awareness of pain and loss, 3) restitution. Acute period: 4-8 weeks, usual resolution: 1 year. (A) She has already moved through the stages of the grieving process — takes one year (B) She is repressing anger related to her husband’s death — not accurate; second stage: crying, regression (C) She is experiencing shock and disbelief related to her husband’s death — CORRECT: denial first stage; inability to comprehend reality of situation (D) She is demonstrating resolution of her husband’s death — too soon

5. The correct answer is C.
Question: Is the depression normal, or something to be concerned about? (A) The treatment plan is not effective; the patient requires a larger dose of lithium — not accurate (B) This is a normal response to lithium therapy; the patient should continue with the current treatment plan — does not address safety needs (C) This is a normal response to lithium therapy; the patient should be monitored for suicidal behavior — CORRECT: delay of 1-3 weeks before med benefits seen (D) The treatment plan is not effective; the patient requires an antidepressant — normal response

Nurse Test (Foundation of Nursing)
January 19, 2010 By admin 4 Comments 1. The most important nursing intervention to correct skin dryness is: a. Avoid bathing the patient until the condition is remedied, and notify the physician b. Ask the physician to refer the patient to a dermatologist, and suggest that the patient wear home-laundered sleepwear c. Consult the dietitian about increasing the patient’s fat intake, and take necessary measures to prevent infection d. Encourage the patient to increase his fluid intake, use nonirritating soap when bathing the patient, and apply lotion to the involved areas

2. When bathing a patient’s extremities, the nurse should use long, firm strokes from the distal to the proximal areas. This technique: a. Provides an opportunity for skin assessment b. Avoids undue strain on the nurse c. Increases venous blood return d. Causes vasoconstriction and increases circulation 3. Vivid dreaming occurs in which stage of sleep? a. Stage I non-REM b. Rapid eye movement (REM) stage c. Stage II non-REM d. Delta stage 4. The natural sedative in meat and milk products (especially warm milk) that can help induce sleep is: a. Flurazepam b. Temazepam c. Tryptophan d. Methotrimeprazine 5. Nursing interventions that can help the patient to relax and sleep restfully include all of the following except: a. Have the patient take a 30- to 60-minute nap in the afternoon b. Turn on the television in the patient’s room c. Provide quiet music and interesting reading material d. Massage the patient’s back with long strokes 6. Restraints can be used for all of the following purposes except to:

a. Prevent a confused patient from removing tubes, such as feeding tubes, I.V. lines, and urinary catheters b. Prevent a patient from falling out of bed or a chair c. Discourage a patient from attempting to ambulate alone when he requires assistance for his safety d. Prevent a patient from becoming confused or disoriented 7. Which of the following is the nurse’s legal responsibility when applying restraints? a. Document the patient’s behavior b. Document the type of restraint used c. Obtain a written order from the physician except in an emergency, when the patient must be protected from injury to himself or others d. All of the above 8. Kubler-Ross’s five successive stages of death and dying are: a. Anger, bargaining, denial, depression, acceptance b. Denial, anger, depression, bargaining, acceptance c. Denial, anger, bargaining, depression acceptance d. Bargaining, denial, anger, depression, acceptance 9. A terminally ill patient usually experiences all of the following feelings during the anger stage except: a. Rage b. Envy c. Numbness d. Resentment 10. Nurses and other health care provides often have difficulty helping a terminally ill patient through the necessary stages leading to acceptance of death. Which of the following strategies is most helpful to the nurse in achieving this goal?

a. Taking psychology courses related to gerontology b. Reading books and other literature on the subject of thanatology c. Reflecting on the significance of death d. Reviewing varying cultural beliefs and practices related to death 11. Which of the following symptoms is the best indicator of imminent death? a. A weak, slow pulse b. Increased muscle tone c. Fixed, dilated pupils d. Slow, shallow respirations 12. A nurse caring for a patient with an infectious disease who requires isolation should refers to guidelines published by the: a. National League for Nursing (NLN) b. Centers for Disease Control (CDC) c. American Medical Association (AMA) d. American Nurses Association (ANA) 13. To institute appropriate isolation precautions, the nurse must first know the:

a. Organism’s mode of transmission b. Organism’s Gram-staining characteristics c. Organism’s susceptibility to antibiotics d. Patient’s susceptibility to the organism 14. Which is the correct procedure for collecting a sputum specimen for culture and sensitivity testing? a. Have the patient place the specimen in a container and enclose the container in a plastic bag

b. Have the patient expectorate the sputum while the nurse holds the container c. Have the patient expectorate the sputum into a sterile container d. Offer the patient an antiseptic mouthwash just before he expectorate the sputum 15. An autoclave is used to sterilize hospital supplies because: a. More articles can be sterilized at a time b. Steam causes less damage to the materials c. A lower temperature can be obtained d. Pressurized steam penetrates the supplies better 16. The best way to decrease the risk of transferring pathogens to a patient when removing contaminated gloves is to: a. Wash the gloves before removing them b. Gently pull on the fingers of the gloves when removing them c. Gently pull just below the cuff and invert the gloves when removing them d. Remove the gloves and then turn them inside out 17. After having an I.V. line in place for 72 hours, a patient complains of tenderness, burning, and swelling. Assessment of the I.V. site reveals that it is warm and erythematons. This usually indicates: a. Infection b. Infiltration c. Phlebitis d. Bleeding 18. To ensure homogenization when diluting powdered medication in a vial, the nurse should: a. Shake the vial vigorously b. Roll the vial gently between the palms

c. Invert the vial and let it stand for 1 minute d. Do nothing after adding the solution to the vial 19. The nurse is teaching a patient to prepare a syringe with 40 units of U-100 NPH insulin for self-injection. The patient’s first priority concerning self-injection in this situation is to: a. Assess the injection site b. Select the appropriate injection site c. Check the syringe to verify that the nurse has removed the prescribed insulin dose d. Clean the injection site in a circular manner with and alcohol sponge 20. The physician’s order reads “Administer 1 g cefazolin sodium (Ancef) in 150 ml of normal saline solution in 60 minutes.” What is the flow rate if the drop factor is 10 gtt = 1 ml? a. 25 gtt/minute b. 37 gtt/minute c. 50 gtt/minute d. 60 gtt/minute 21. A patient must receive 50 units of Humulin regular insulin. The label reads 100 units = 1 ml. How many milliliters should the nurse administer? a. 0.5 ml b. 0.75 ml c. 1 ml d. 2 ml 22. How should the nurse prepare an injection for a patient who takes both regular and NPH insulin? a. Draw up the NPH insulin, then the regular insulin, in the same syringe b. Draw up the regular insulin, then the NPH insulin, in the same syringe

c. Use two separate syringe d. Check with the physician 23. A patient has just received 30 mg of codeine by mouth for pain. Five minutes later he vomits. What should the nurse do first? a. Call the physician b. Remedicate the patient c. Observe the emesis d. Explain to the patient that she can do nothing to help him 24. A patient is characterized with a #16 indwelling urinary (Foley) catheter to determine if: a. Trauma has occurred b. His 24-hour output is adequate c. He has a urinary tract infection d. Residual urine remains in the bladder after voiding 25. A staff nurse who is promoted to assistant nurse manager may feel uncomfortable initially when supervising her former peers. She can best decrease this discomfort by: a. Writing down all assignments b. Making changes after evaluating the situation and having discussions with the staff. c. Telling the staff nurses that she is making changes to benefit their performance d. Evaluating the clinical performance of each staff nurse in a private conference 1. Answer – D. Dry skin will eventually crack, ranking the patient more prone to infection. To prevent this, the nurse should provide adequate hydration through fluid intake, use nonirritating soaps or no soap when bathing the patient, and lubricate the patient’s skin with lotion. Bathing may be limited but need not be avoided entirely. The attending physician and dietitian may be consulted for treatment, but homelaundered items usually are not necessary.

2. Answer – C. Washing from distal to proximal areas stimulates venous blood flow, thereby preventing venous stasis. It improves circulation but does not result in vasoconstriction. The nurse can assess the patient’s condition throughout the bath, regardless of washing technique, and should feel no strain while bathing the patient. 3. Answer – B. Other characteristics of rapid eye movement (REM) sleep are deep sleep (the patient cannot be awakened easily), depressed muscle tone, and possibly irregular heart and respiratory rates. Non-REM sleep is a deep, restful sleep without dreaming. Delta stage, or slow-wave sleep, occurs during non-REM Stages III and IV and is often equated with quiet sleep. 4. Answer – C. Tryptophan is a natural sedative; flurazepam (Dalmane), temazepam (Restoril), and methotrimeprazine (Levoprome) are hypnotic sedatives. 5. Answer – A. Napping in the afternoon is not conductive to nighttime sleeping. Quiet music, watching television, reading, and massage usually will relax the patient, helping him to fall asleep. 6. Answer – D. By restricting a patient’s movements, restraints may increase stress and lead to confusion, rather than prevent it. The other choices are valid reasons for using restraints. 7. Answer – D. When applying restraints, the nurse must document the type of behavior that prompted her to use them, document the type of restraints used, and obtain a physician’s written order for the restraints. 8. Answer – C. Kubler-Ross’s five successive stages of death and dying are denial, anger, bargaining, depression, and acceptance. The patient may move back and forth through the different stages as he and his family members react to the process of dying, but he usually goes through all of these stages to reach acceptance. 9. Answer – C. Numbness is typical of the depression stage, when the patient feels a great sense of loss. The anger stage includes such feelings as rage, envy, resentment, and the patient’s questioning “Why me?” 10. Answer – C. According to thanatologists, reflecting on the significance of death helps to reduce the fear of death and enables the health care provider to better understand the terminally ill patient’s feelings. It also helps to overcome the belief that medical and nursing measures have failed, when a patient cannot be cured. 11. Answer – C. Fixed, dilated pupils are sign of imminent death. Pulse becomes weak but rapid, muscles become weak and atonic, and periods of apnea occur during respiration. 12. Answer – B. The Center of Disease Control (CDC) publishes and frequently updates guidelines on caring for patients who require isolation. The National League of

Nursing’s (NLN’s) major function is accrediting nursing education programs in the United States. The American Medical Association (AMA) is a national organization of physicians. The American Nurses’ Association (ANA) is a national organization of registered nurses.

13. Answer – A. Before instituting isolation precaution, the nurse must first determine the organism’s mode of transmission. For example, an organism transmitted through nasal secretions requires that the patient be kept in respiratory isolation, which involves keeping the patient in a private room with the door closed and wearing a mask, a grown, and gloves when coming in direct contact with the patient. The organism’s Gram-straining characteristics reveal whether the organism is gramnegative or gram-positive, an important criterion in the physician’s choice for drug therapy and the nurse’s development of an effective plan of care. The nurse also needs to know whether the organism is susceptible to antibiotics, but this could take several days to determine; if she waits for the results before instituting isolation precautions, the organism could be transmitted in the meantime. The patient’s susceptibility to the organism has already been established. The nurse would not be instituting isolation precautions for a noninfected patient. 14. Answer – C. Placing the specimen in a sterile container ensures that it will not become contaminated. The other answers are incorrect because they do not mention sterility and because antiseptic mouthwash could destroy the organism to be cultured (before sputum collection, the patient may use only tap water for nursing the mouth). 15. Answer – D. An autoclave, an apparatus that sterilizes equipment by means of hightemperature pressured steam, is used because it can destroy all forms of microorganisms, including spores. 16. Answer – C. Turning the gloves inside out while removing them keeps all contaminants inside the gloves. They should than be placed in a plastic bag with soiled dressings and discarded in a soiled utility room garbage pail (double bagged). The other choices can spread pathogens within the environment. 17. Answer – C. Tenderness, warmth, swelling, and, in some instances, a burning sensation are signs and symptoms of phlebitis. Infection is less likely because no drainage or fever is present. Infiltration would result in swelling and pallor, not erythema, near the insertion site. The patient has no evidence of bleeding. 18. Answer – B. Gently rolling a sealed vial between the palms produces sufficient heat to enhance dissolution of a powdered medication. Shaking the vial vigorously can break down the medication and alter its pharmacologic action. Inverting the vial or leaving it alone does not ensure thorough homogenization of the powder and the solvent.

19. Answer – C. When the nurse teaches the patient to prepare an insulin injection, the patient’s first priority is to validate the dose accuracy. The next steps are to select the site, assess the site, and clean the site with alcohol before injecting the insulin. 20. Answer – A. 25 gtt/minute 21. Answer – A. 0.5 ml 22. Answer – B. Drugs that are compatible may be mixed together in one syringe. In the case of insulin, the shorter-acting, clear insulin (regular) should be drawn up before the longer-acting, cloudy insulin (NPH) to ensure accurate measurements. 23. Answer – C. After a patient has vomited, the nurse must inspect the emesis to document color, consistency, and amount. In this situation, the patient recently ingested medication, so the nurse needs to check for remnants of the medication to help determine whether the patient retained enough of it to be effective. The nurse must then notify the physician, who will decide whether to repeat the dose or prescribe an antiemetic. 24. Answer – B. A 24-hour urine output of less than 500 ml in an adult is considered inadequate and may indicate kidney failure. This must be corrected while the patient is in the acute state so that appropriate fluids, electrolytes, and medications can be administered and excreted. Indwelling catheterization is not needed to diagnose trauma, urinary tract infection, or residual urine. 25. Answer – B. A new assistant nurse manger should not make changes until she has had a chance to evaluate staff members, patients, and physicians. Changes must be planned thoroughly and should be based on a need to improve conditions, not just for the sake of change. Written assignments allow all staff members to know their own and others responsibilities and serve as a checklist for the manager, enabling her to gauge whether the unit is being run effectively and whether patients are receiving appropriate care. Telling the staff nurses that she is making changes to benefit their performance should occur only after the nurse has made a thorough evaluation. Evaluations are usually done on a yearly basis or as needed.

NIMHANS ENTRANCE TEST QUESTIONS FOR STAFF NURSES
1) Which is not a quality of a self-actualised person? a) Humor b) Dependent c) Good relationship d) Problem centered in approach

2) Anxiety is always present and accompanied by a feeling of dread is termed as; a) Signal anxiety b) Anxiety state c) Free Floating anxiety d) Anxiety trait 3) Which is not a part of the rehabilitation plan of the mentally ill patients? a) Regular medication b) Avoiding crisis support c) Skills training d) Specialized training for vocation 4) Intelligent quotient ( IQ)level of 20-34 indicates? a) Mild mental retardation b) Moderate mental retardation c) Profound mental retardation d) Severe mental retardation 5) Which drug is found to be most useful in the treatment of bipolar affective disorder(BPAD)? a) Chlorpromazine b) Lithium carbonate c) Librium d) Pecitane 6) Persistent recurrence of unwanted and often distressing thoughts is; a) Obsessive compulsive disorder (OCD) b) Depression c) Compulsion d) Obsession

) To maintain adequate cerebral perfusion pressure (CPP), which of the following is true; a) ICP less than 15 mm of Hg, MAP above 50 mm of Hg b) ICP more than 15 mm of Hg, MAP below 50 mm of Hg c) ICP lmore than 25 mm of Hg, MAP below 70 mm of Hg d) ICP lmore than 30 mm of Hg, MAP below 40 mm of Hg 2) Which of the following is malignant tumor?

a) Glicoma b) Neurofibroma c) Thyroid tumor d) Kaposi's sacroma 3) Haemophilia type A & b are: a) Autosomal dominant traits b) Dominant sex linked traits c) Autosomal recessive traits d) Recessive sex linked traits 4) When is the most appropriate time for a women to perform breast self examination? a) The first day of the menstrual period b) One week after the start of the menstrual period c) Just before the time of intercourse d) Any convenient time of the month 5) A client receiving recieving rifampicin should be taught that the effect of the drug could cause; a) Reddish orange color of urine, saliva and sputum b) Eighth cranial nerve damage c) Brain d) vestibular dysfunction 6) Which of the following is an unlikely assessment for a client with a diagnosis of hepatic encephalopathy; a) Muscular pain b) Muscle twitching c) Exophthamos d) Decreased level of consciousnes

1) Which is the most important nurse's responsibility on the day of surgery? a) Remove all clothing's b) Send blood for investigations c) Teach deep breathing exercises d) Administer Pre-anesthetic medications 2) The purpose of post-operative deep deep breathing and coughing exercises are to: a) Reduce pain b)Prevent wound infection c) Prevent Apnea d) prevent atelectasis 3) Soft systolic ejection murmur heard in elder person is commonly due to: a) Sclerotic changes of aortic leaflets b) left ventricle become smaller

c) Decreased elasticity & widening of aorta d) Increased Blood pressure 4) Gerontology is the study of a) Old age b) care of old c) Diseases related to aging d) Aging process 5) Which of the following manifestations would a nurse expect to observe in a patient immediately following a tonic-clonic generalized seizure? a) Apnea b) Tachypnoea c) Lethargy d) Hypersalivation 6) The precipitating factor for myasthenic crisis is; a) Increase intake of fatty acids b) Omitted doses of medication c) Weight lifting d) Excess medication

When caring for a client with continuous bladder irrigation, the nurse should, a) Record output every hour b) Monitor urinary speific gravity c) Subtract irrigant from output to determine urine volume d) Include irrigating solution in any 24 hour urine tests order * The nurse can prevent the contamination from Mrs. Jacinta's retention catheter by: a) Irrigating the catheter b) Perineal cleansing c) Encouraging fluids d) Cleansing around the meatus periodically * The major reasons for treating severe emotional disorders with tranquilizers is to; a) Reduce the neurotic syndrome b) Prevent secondary complication c) Make the client amenable to physiotherapy

d) Prevent destructiveness by the client * The most important factor in rehabilitation of a client addicted to alcohol is; a) The Clients emotional or motivational readiness b) The availability of community resources c) The qualitative level of the clients physical state d) The accepting attitude of client's family * Which of the following activities would cause her a risk in the increase of intracranial pressure? a) Exercise b) Coughing c) Turning d) Sleeping * Which of the following drug may be given to reduce increase intracranial pressure? a) Mannitol b) Scopalamine c) Lanoxin d) Calmpose * Which of the following is a form of active, focused, emotional environmental first aid for patients in crisis? a) Attitude therapy b) Psychotherapy c) Re motivation technique d) Crisis intervention * The major treatment for ascities calls for; a) Increased potassium b) High protein c) Restricted fluids d) Restricted sodium * The major influence of eating habits of the early school-aged child is;

a)Spoon feeding b) Availability of food selections c) Smell and appearance of food d) Example of parents at meal time 1) Basic Principles of fundamentals of nursing a) Cleaning b) Disinfection c) Sterilization d) All the above 2) Example for second class lever a) Scissors b) Trolley c) O2 tank carrier d) None of the above 3) Radio active cobalt is used a) For surgery b) For Drug c) Locally treating d) None of the above 4) SOS means a) Once a day b) 4 times a day c) If necessary d) At night 5) Elements of primary health care a) Promotion of food supply & proper nutrition b) Adequate supply of safe water & sanitation

c) Maternal and child health care d) All the above 6) The technique is used to open O2 cylinder a) Clock wise b) Anti clock wise c) Upwards d) Downwords

* Parentral administration of drug is
a) IM b) IV c) Subcutaneous d) all the above

* Nursing diagnosis means
a) Equal to doctor diagnosis b) Different from medical diagnosis c) Done by the docto d) None of the above

* How will you identify the sensitivity of the drug
a) IM dose b) IV dose c) Test dose d) Lethal dose

* Every trained nurse must be registered in

a) Medical council b) Nursing council c) Health council d)None of the above

* Rh factor contains
a) Antigen A b) Antigen C c) Antigen D d) Antigen B

* Responsibilities of graduate Nurses
a) Patient care b) Doctor c) Hospital d) Director

* The drugs are used for steam inhalations
a) TR Benzine b) Eucalyptus c) Menthol d) Camphor e) All the above

* The steps of Nursing process
a) Assessment b) Planning c) Implementation d) Evaluation

Ischial tuberosity helps for a) Normal labour b) Normal walking c) Normal standing

d) Bearing of body weight while sitting

* Injury to pelvic floor
a) Sometimes occurs in first stage of labour b) Sometimes occur in second stage of labour c) Does not arise d)During first and second stage of labour

* Morulla is a ball of cells
a) Indication of conception b) It is a benign tumor c) Indication of ca. cervix d) It is a cavity filled with abnormal cells

* The fetal heart pumps an amount of blood through the placenta per minute
a) 300 ml b) 1000 ml c) 800 ml d)500 ml

* Signs of fetal death
a) The uterus increases in the size b) A brownish discharge per vagina c) Fetal heart sounds present

d) Fetal moments absent

* Largest diameter of the fetal skull is
a) Sub mento vertical b) Mento Occipital c) Mento vertical d)Sub occipito frontal

* The nurse takes the following pledge
a) Nightingale pledge b) Christian pledge c) Hippocrates pledge d) Catholic pledge

* A brief description of an accident
a) Work report b) Anecdotal report c) Self development d) All the above

* Number of patients in the hospital at mid night or before 6 a.m. is called
a) Time report b) Work report c) Lencess report d) None of the above

* Change nurse is

a) Nursing superintendent b) Head nurse c) Staff nurse d) ANM

* Most important records in the hospitals
a) Patients clinical report b) Treatment, admission and discharge record c) Ward equipment record d) All the above

* The following is responsible for 24 hours nursing care in the hospital
a) Student nurse b) Staff nurse c) Head nurse d) Nursing superintendent

* Organized institute for care of sick & injured
a) Hospital b) Blood bank c) Office d) None of the above

* Well planned schedule of duty hours
a) Time schedule b) Duty Roaster c) Rotation d) None of the above

• Prevention of psychological reaction is in
a) Primary prevention b) Secondary prevention c) Tertiary prevention d) All the above

* In electro convulsive therapy (BPST -Brief pulse stimulating therapy) 90 to 20
volts of 150 cycles alternating current passed through electrodes which are applied bi-temporally for a) 0.3 to 0.7 seconds b) 10-15 seconds c) 1 minute d) 16 seconds

* The illness of melancholia is characterized by symptoms of
a) Sadness of mood b) Poverty of ideas c) None of the above

d) None of the above

* When a person having abnormally aggressive, and antisocial behavior is known
as a) Psychopathy b) Socio therapy c) Physiotherapy d) None of the above

* Indications for psychotherapy
a) Anxiety neurosis b) Psychosomatic illness c) Schizophrenia, depression, mania d) All the above

* Unexplained and irrational morbid tears about animate and/or in animate objects
is known as a) Tension b) Ideopathy c) Phobias d) None of the abo Exact heart rate found at a) radial artery b) Apex of the heart c) Temporal vein d) Femoral vein

* Specific gravity of urine

a) 1.010-1.020 b) 1.000-1.005 c) 1.030-1.010 d) 800-1000

* The temperature of retained enema
a) 100 F b) 90 F c) 98.6 F d) 92 F

* The correct position for giving enema
a) Left lateral b) Sims c) prone d) Sitting

* The route of insulin injection
a) Subcutaneus b) IV c) IM d)Intrathecal

* The universal blood donor is
a) AB b) B c) A d) O

* Cardiac rest used for

a) Asthma b) Cirrhosis of liver c) Pain abdomen d) Cardiac patient

* Immediate treatment of dog bite
a) Send the hospital b) Application of antiseptic c) Wash the wound with soap and water d) Suturing

* Dislocation is diagnosed by the features
a) Pain over the joint b) Swelling of the joint c) Deformity d) All the above

* HS means
a) Thrice a day b) At bed time c) Twice a day d) None of the above * One pint equal to a) 30 ounce b) 400 ml c) 20 ounce or 500 ml d)None of the above

* Indian Nursing council was established in the year
a) 1965 b) 1918 c) 1960 d) 1950 The incidents of schizophrenia is very high in a) Normal child b) Uniovular twins c) Bi ovular twins d) None of the above

* How can you recognize a mentally ill person
a) Rowdy, violent b) Anxiety, tension c) Abusive, homicidal behaviors d) All the above

* The branch of medicine which deals with the diagnosis, treatment and prevention of
mental illness. a) Psychology b) Sociology c) Psychiatry d)None of the above

The spinal nerves are a) 30 pairs b) 31 pairs

c) 41 pairs d) 20 pairs * Ovulation day is a) 14 days of menses b) 10th day of menses c) 9th day of menses d) 20th day of menses * Number of milk teeth a) 20 b) 30 c) 32 d) 28 * Shape of the body of thoracic vertebrae a) kidney shape b) Heart shape c) Bean shape d) Pea nut shape * Smallest bone present in the body a) Head b) Leg c) Ear d) Hand * Cardiac sphincter is situated at a) Heart b) Cardiam c) End of esophagus d) End of stomach

* Voice produced by a) Pharynx b) Larynx c) Vocal cords d)Thyroid cartilage * Function of the blood a) Carry carbon dioxide to the blood b) Carry nutrients to all parts of the body c) Carry antibodies to sites of infection d) All the above

* Which is the biggest artery in the body
a) Brachial artery b) Aorta c) Femoral artery d)Tibial artery

* Life span of RBC
a) 100 days b) 120 days c) 1 year d) 60 days * The heart is situated in the a) abdominal cavity b) Thoracic cavity c) Pelvic cavity d)Cranial cavity

* Normal sperm count should be
a) 20 million or above/ml b) 5 million or less c) 50 thousands d) None of the above

* Sex hormones are
a) Thyroxine b) Insulin c) Growth Hormone d)Testosterone

* Antero posterior diameter of the gynaecoid pelvic outlet is
a) 11 cm b) 10 cm c) 13 cm d) 12 cm

* Difficult labour is called as
a) Dyspepsia b) Dysparenia c) Dyspnoea d)Dystocia

* Quickening is first felt by the mother at
a) 10 weeks b) 24 weeks c) 20 weeks

d) 36 weeks

* Ovum has the pair of sex chromosomes
a) XXX b) XY c) XX d)None of the above

* Anaemia is commonly seen in the following women
a) Child bearing women b) Multi parity women c) Lower income group d) all the above

* The denominator in face presentation
a) occiput b) Sacrum c) Mentum d)Acromion process

* During 3rd week of pregnancy
a) The ovum is without villi b) The ovaries covered with chorionic villi c) The ovum is absent d) None of the above

* Fertilization of ovam & sperm takes place
a) In uterus b) Cervix c) Vagina

d) Ampulla of fallopian tube

* The vaginal vault is devided into
a) 6 fornices b) 3 fornices c) 4 fornices d) 8 fornices

* Pelvic bones are
a) Femur b) Radius c) Sacrum and coccyx d) Humerus

* Gynaecoid pelvis has
a) Oval shape brim b) Wedge shaped c) Flat shape brim d) None of the above

* Infective hepatitis virus is present in
a) Saliva b) Sputum c) Faces & blood d) All above

* Coal dirt causes

a) Anthracosis b) Silicosis c) Bagassosis d) Byssinosis

* The stools of cholera are
a) Green b) yellow c) Rice water d) Normal

* Rabies is transmitted by the bite of
a) Anopheles mosquito b) Culex mosquito c) Flies d) A rabid dog

* Vaccine produces
a) Passive immunity b) Active immunity c) Active and passive immunity d) All above

* Important objectives of health services are
a) Prevention of diseases b) Promotion of health c) Cure from the diseases d) Above all

* Audio visual aids are classified in to
a) Auditory b) Visual c) Combined d) All above

* Year of 1981 was declares as year of
a) Immunization b) Handicapped c) youth d) Nurses

* Copper T is contra indicated during
a) Pregnancy b) Pelvic infections c) Heart diseases d) All above

* Lung volumes may be determined by
a) Barometer b) Spirometry c) Sphigmomanometer d) Lactometer

* Echocardiography is done for the following
a) To assess the size of the cardiac chambers b) To assess the valvular function

c) To detect septal defects d) All the above

* Definition of 'MI' is
a) Irritation of the covering of the heart b) Atherosclerosis of the coronary arteries c) Distraction of mitral valve d) Death of heart muscle

* Deficiency of sodium in the blood is known as
a) Hyper natraemia b) Hyper kalemia c) Hypo natraemia d) Hypo kalemia

1. How long should a certified nursing assistant wait to take a patient’s temperature if the patient has just ingested a cold beverage? 2. What unit of measurement should be used to gauge an amount of a patient’s urine? 3. What maneuver should be performed on a patient with a blocked airway? 4. What are range of motion exercises? 5. If a patient experiences weakness on one side of their body, should you start dressing them on that side? Answers: 1. 10-20 minutes 2. cc’s 3. Heimlich 4. Exercises that move every joint and muscle 5. …the weak side

Sponsor Documents

Or use your account on DocShare.tips

Hide

Forgot your password?

Or register your new account on DocShare.tips

Hide

Lost your password? Please enter your email address. You will receive a link to create a new password.

Back to log-in

Close